Vision Ias Prelims

  • Uploaded by: CodMw
  • 0
  • 0
  • January 2021
  • PDF

This document was uploaded by user and they confirmed that they have the permission to share it. If you are author or own the copyright of this book, please report to us by using this DMCA report form. Report DMCA


Overview

Download & View Vision Ias Prelims as PDF for free.

More details

  • Words: 20,743
  • Pages: 56
Loading documents preview...
1/25/2016

Vision IAS Prelims

Introduction (introduction.php?test_id=1763&package_id=1276&compare_with=3&qid=1)

Take Test (objtest.php?test_id=1763&package_id=1276&compare_with=3)

Result

Post Test Analysis

Module Performance Question Paper & Answer Format (question_paper_op.php?test_id=1763&mode=1&package_id=1276)

Discussion Video (discussion_video.php?test_id=1763&package_id=1276&mode=2)

Home (pt_list_2.php)

Why Post Test Analysis The purpose of writing a test is not limited to getting a score. You cannot be assured of realizing your potential by simply knowing the answers of all the mock tests. The only guaranteed way to succeed is to analyse your performance. The tendency to forget about the test after appearing is detrimental at best and disastrous at worst. The right way is to pay attention to what went wrong and what worked. Each test that you appear, must provide you a learning experience for improving your capabilities and more importantly, serve as a guide to help you translate that learning into practice in future tests. Research suggests that explicitly teaching students to analyse their test performance can help them to better assess the understanding of their own cognitive process i.e. metacognitive awareness. We, at VISIONIAS, have evolved a comprehensive methodology to assess your various aspects of your test taking ability. Our method will not only take you to the next level of preparation but also give you the distinct competitive edge that is required in the current scheme of examination. Hence, we urge you to follow this process with all sincerity and let us help you to analyse your performance acutely.                                                       

Question

Options

1. How prepared were you for the test?(0­10) Here you need to mention your level of preparedness for the test as per your own assessment. 

Select

Know More

2. How many month did you prepare for UPSC Exam (0­99) Here, one should mention the number of months, he/she has been preparing for the civil service examination.

3. Rate the satisfaction of your preparation (0­10) This reflects your satisfaction with your preparation after you have appeared in the test.

Select

Know More

4. Did you refer to standard Book ? Yes  No Choose ‘Yes’ if you referred to the recommended standard books like Spectrum or Bipan Chandra for History, Laxmikant’s polity etc in the recent past. http://visionias.in/student_open/pt/pta_form.php?test_id=1763&package_id=1276&compare_with=3

1/56

1/25/2016

Vision IAS Prelims

Else choose ‘No’.

5. Did you refer advance book ? Apart from the standard text books, there are often a good number of books meant for further in­depth and thorough study of the subject and topics appearing in the test. E.g. DD Basu in polity, Strahler or Savindra Singh in Geography, etc. If you have referred to these sources for the test prep or at least in the recent past, you may Choose ‘Yes’. Else Choose ‘No’.

Yes  No

6. Did you refer Previous year UPSC paper? Yes  No Choose ‘Yes’ if you have referred to the past UPSC papers comprehensively in past couple of months, for at least the topics relevant for the test. Else Choose ‘No’

7. Did you refer NCERT ? NCERT books are considered to be fundamental to UPSC preparation and are a treasure trove of relevant information for the aspirants. Year after year, we see questions appearing from NCERT in UPSC prelims and mains. Choose ‘Yes’ if you referred to the NCERTs, at least for the topics covered in the test. Else Choose ‘No’

Yes  No

Attempted type:  All Q. No. Question With reference to the growth of Sufi Movements in the religious history of medieval India, consider the following statements: 1.  It rose initially against the protest of growing materialism of the Caliphate as a religious and political institution. 2.  Sufis were critical of the dogmatic interpretation of Quran by the theologians. 3.  Sufis of all order defied 'sharia' laws.

Answer, Difficulty and Nature a Medium Factual(F) Detailed Analysis (question_analysis.php? test_id=1763&package_id=1276&compare_with=3&qid=22913&et1=1&et2=9)

Which of the statements given above is/are correct? (a) 1 and 2 only (b) 2 and 3 only (c ) 3 only 1 http://visionias.in/student_open/pt/pta_form.php?test_id=1763&package_id=1276&compare_with=3

Your Answer Exp. and Marks a Correct :2 Marks  Time:80 Sec.

Post Test Analysis Statement 1 is correct: In the early centuries of Islam, a group of religious minded people called the 'Sufis' turned into ascetism and mysticism in protest against the growing materialism of the Caliphate as a religious and political institution. Statement 2 is correct: The Sufis were critical of the dogmatic definitions & scholastic methods of interpreting the Quran and 'Sunna' (traditions of Prophet) adopted by the theologians. Instead they laid emphasis on seeking salvation through intense

Attempted

Guessed between two/ eliminated two Guessed between three/ eliminated one Blind Guess Knew the answer / solved the question

2/56

1/25/2016

Vision IAS Prelims

(d) 1, 2 and 3

The term 'Polar' in Polar Satellite Launch Vehicle (PSLV) signifies that (a) the PSLV satellite orbits based on the position of pole star. (b) the PSLV satellite orbits the earth from pole­to­pole.

devotion & love for God.  Statement 3 is incorrect: Some Sufis initiated movements against radical interpretations of sufi ideals. They ignored rituals & practised extreme ascetism, observed celibacy, etc. they were known by different names like ­  'Qalandars', 'Madaris', 'Malangs', ' Haidaris', etc. Because of their deliberate defiance of sharia they were known as 'be­sharia' , in contrast to those Sufis who complied with the sharia (ba­ sharias).

b Medium Analytical(A) Detailed Analysis (question_analysis.php? test_id=1763&package_id=1276&compare_with=3&qid=22912&et1=1&et2=9)

(c) PSLV satelite is launched from North pole because of low air resistance. (d) PSLV satellites use Polaroids for remote sensing and image capture. 

c  Incorrect  :­0.67 Marks  Time:37 Sec.

PSLV launches the satellites in polar orbits. That is, the satellites which would revolve around the earth from south pole to north pole(pole to pole) will be launched by PSLVs.   The PSLV is one of world's most reliable launch vehicles. It is used for delivering various satellites to Low Earth Orbits. It is designed mainly to deliver the "earth­observation" or "remote­ sensing" satellites with lift­off mass of up to about 1750 Kg to Sun­Synchronous circular polar orbits of 600­900 Km altitude.

Attempted

Guessed between two/ eliminated two Guessed between three/ eliminated one Blind Guess Knew the answer / solved the question

Few important points about PSLV and GSLV:

2

http://visionias.in/student_open/pt/pta_form.php?test_id=1763&package_id=1276&compare_with=3

1.  The PSLV and GSLV are two rocket launch systems developed by ISRO, to launch satellites into orbit. 2.  The PSLV is older than the GSLV 3.  The GSLV has a much greater load capacity than the PSLV. While the PSLV can only lift slightly over a ton of payload, the GSLV is capable of lifting more than double that with a rated capacity of 2 to 2.5 tons. 4.  The GSLV use cryogenic fuel while the PSLV doesn't 5.  The PSLV has 4 stages 3/56

1/25/2016

Vision IAS Prelims

that alternate between solid and liquid fuels while the GSLV has three stages with the only the first stage having solid fuel. 6.  The GSLV has 4 liquid boosters while the PSLV has 6 solid boosters.

Which of the following states of India have regions of rainfall exceeding 200cm as well as the regions with less than 50cm of rainfall? 1.  2.  3.  4. 

Maharashtra Rajasthan West Bengal Karnataka

b Medium Analytical(A) Detailed Analysis (question_analysis.php? test_id=1763&package_id=1276&compare_with=3&qid=22911&et1=1&et2=9)

a  Incorrect  :­0.67 Marks  Time:59 Sec.

Western Ghats in Karnataka and Maharashtra receive more than 200cm of rainfall. Deccan plateau regions of Maharashtra and Karnataka receive less than 50cm of rainfall. West Bengal has no regions with less than 50cm of rainfall. Rajasthan has no regions with more than 200cm of rainfall.

not attempted

Photochemical smog is a condition that develops when primary pollutants (oxides of nitrogen and volatile organic compounds created from fossil fuel combustion) interact under the influence of sunlight to produce a mixture of hundreds of different and hazardous chemicals known as secondary pollutants. Nitrogen Oxides, Volatile Organic Compounds

Select the correct answer using the code given below. (a) 1 and 3 only (b) 1 and 4 only 3

Attempted

Guessed between two/ eliminated two Guessed between three/ eliminated one Blind Guess Knew the answer / solved the question

(c) 2 and 4 only (d) 1, 3 and 4 only

Which of the following chemical pollutants is/are present in Photochemical Smog? 1.  2.  3.  4. 

Nitrogen Oxides Volatile Organic Compounds (VOC) Ozone Peroxyacetyl Nitrates (PAN)

d Difficult Factual(F) Detailed Analysis (question_analysis.php? test_id=1763&package_id=1276&compare_with=3&qid=22910&et1=1&et2=9)

Select the correct answer using the code given below. 4

(a) 1 and 2 only (b) 1, 2 and 3 only (c) 2 and 3 only (d) 1, 2, 3 and 4

http://visionias.in/student_open/pt/pta_form.php?test_id=1763&package_id=1276&compare_with=3

Unattempted

Absolutely no idea / Could not comprehend the question Not sure of the right answer Inability to recall technique/method Fatigue: not able to concentrate Guessed 4/56

1/25/2016

Vision IAS Prelims

(VOC), Ozone and Peroxyacetyl Nitrates (PAN) are the major toxic constituents of photochemical smog.

Recently the Cabinet Committee on Economic Affairs (CCEA) has approved the creation of a buffer stock for pulses. Which of the following agencies would be involved in procurement? 1.  Food Corporation of India (FCI) 2.  National Agricultural Cooperative Marketing Federation of India Ltd. (NAFED) 3.  Small Farmers Agribusiness Consortium (SFAC)

d

not attempted

Medium Factual(F) Detailed Analysis (question_analysis.php? test_id=1763&package_id=1276&compare_with=3&qid=22909&et1=1&et2=9)

Select the correct answer using the codes given below. (a) 1 only 5

(b) 1 and 2 only (c) 2 and 3 only (d) 1, 2 and 3

Consider the following features about an Indian painting: 1.  Traditionally, done by womenfolk on mud walls, it is now also painted on cloth, handmade paper and canvas. 2.  Its main theme revolves around Hindu deities. 3.  Generally, empty spaces are filled with various motifs.

b Medium Factual(F) Detailed Analysis (question_analysis.php? test_id=1763&package_id=1276&compare_with=3&qid=22908&et1=1&et2=9)

Which of the following paintings is characterized by the above mentioned features? (a) Kangra Painting (b) Mithila Painting http://visionias.in/student_open/pt/pta_form.php?test_id=1763&package_id=1276&compare_with=3

b Correct :2 Marks  Time:23 Sec.

The Cabinet Committee on Economic Affairs (CCEA) has given its approval for creation of buffer stock of pulses. The buffer stock will be created in current year itself. It has approved procurement of about 50,000 ton pulses from the kharif crop 2015­16 and one lakh ton out of arrivals of rabi crop of 2015­16. Procurement of pulses will be done at market prices through Food Corporation of India (FCI), National Agricultural Cooperative Marketing Federation of India Ltd. (NAFED), Small Farmers Agribusiness Consortium (SFAC) and any other agency as may be decided. SFAC will undertake procurement through Farmer Producer Organisations. The procurement in kharif and rabi 2015­16 will be done at market price above Minimum Support Price (MSP) out of the Price Stabilisation Fund.

Mithila painting (also known as Madhubani Painting) originated in a small village, known as Maithili, of the Bihar state of India. Themes of the Maithili painting of Bihar revolve around Hindu deities like Krishna, Rama, Lakshmi, Shiva, Durga and Saraswati. The natural themes that are used include the Sun, the Moon and the religious plants like tulsi.

incorrectly

Unattempted

Absolutely no idea / Could not comprehend the question Not sure of the right answer Inability to recall technique/method Fatigue: not able to concentrate Guessed incorrectly

Attempted

Guessed between two/ eliminated two Guessed between three/ eliminated one Blind Guess Knew the answer / solved the question

5/56

1/25/2016

Vision IAS Prelims

(c) Kalamkari Painting

The painting was traditionally done on freshly plastered mud walls and floors of huts, but now they are also done on cloth, handmade paper and canvas. One can also find paintings based on scenes from the royal courts and social events, like weddings. If any empty space is left after painting the main theme, it is filled up with the motifs of flowers, animals and birds or geometric designs. Mithila painting, as a domestic ritual activity, was unknown to the outside world until the massive India­Nepal border earthquake of 1934 when the houses and walls tumbled down. Then British colonial officer in Madhubani District, William G. Archer, while inspecting the damage "discovered" the paintings on the newly exposed interior walls of Mithila homes.

(d) Warli Painting

6

Which of the following are classified under the Right to Freedom of Religion? 1.  Prohibition of discrimination on grounds of religion. 2.  Freedom of conscience and free profession of religion. 3.  Protection of culture of minorities. 4.  Freedom to manage religious institutions and affairs. 7

c Easy Factual(F) Detailed Analysis (question_analysis.php? test_id=1763&package_id=1276&compare_with=3&qid=22907&et1=1&et2=9)

Select the correct answer using the code given below. (a) 1 and 2 only (b) 1, 2 and 4 only (c) 2 and 4 only (d) 2, 3 and 4 only

http://visionias.in/student_open/pt/pta_form.php?test_id=1763&package_id=1276&compare_with=3

c Correct :2 Marks  Time:78 Sec.

1st and 3rd statements are incorrect as they pertain to Article 15 and 29 respectively. Right to Freedom of Religion consists of the following Articles (25­28): Article 25 ­ Freedom of conscience and free profession, practice and propagation of religion. Article 26­ Freedom to manage religious affairs. Article 27­ Freedom from payments of taxes for promotion of any religion. Article 28­ Freedom as to attendance at religious instruction or religious worship in certain educational institutions

Attempted

Guessed between two/ eliminated two Guessed between three/ eliminated one Blind Guess Knew the answer / solved the question

6/56

1/25/2016

Vision IAS Prelims

Consider the following Government Acts/policies: 1.  Shram Suvidha portal 2.  Arbitration and Conciliation (Amendment) Act, 2015 3.  Retrospective Taxation 

b Easy Analytical(A) Detailed Analysis (question_analysis.php? test_id=1763&package_id=1276&compare_with=3&qid=22906&et1=1&et2=9)

b Correct :2 Marks  Time:40 Sec.

Shram Suvidha Portal under ministry of labour and employment facilitates ease of reporting at one place for various Labour Laws, consolidated information of Labour Inspection and its enforcement. It is one of the various steps taken by government of India regarding labor reforms to improve ease of doing business. Recently Arbitration and Conciliation (Amendment) Act, 2015 and Commercial Courts, Commercial Division and Commercial Appellate Division of High Courts Act, 2015 was passed by the parliament for expeditious settlement of commercial disputes that would improve ease of doing business in the country. Retrospective taxation will hurt the interests of investors and thus will not improve ease of doing business. Hence B is the correct answer. 

a Correct :2 Marks  Time:18 Sec.

The Global Competitiveness Report (GCR) is a yearly report published by the World Economic Forum. The World Economic Forum (WEF) is a Swiss nonprofit foundation, based in Cologny, Geneva. India has been placed at 55th position on 2015 Global Competitiveness Index of the world's most competitive economies. India moved up 16 positions in this edition of index as compared to 71th position in 2014.



Small savings (Post­Office

Which of the initiatives given above will improve ease of doing business in India? (a) 1  only (b) 1 and 2 only (c) 3 only (d) 1, 2 and 3  8

Which one of the following publishes the 'Global Competitiveness Index' report periodically? (a) World Economic Forum (b) The World Bank

a Medium Factual(F) Detailed Analysis (question_analysis.php? test_id=1763&package_id=1276&compare_with=3&qid=22905&et1=1&et2=9)

(c) IMF 9

(d) US Federal Bank

d http://visionias.in/student_open/pt/pta_form.php?test_id=1763&package_id=1276&compare_with=3

Attempted

Guessed between two/ eliminated two Guessed between three/ eliminated one Blind Guess Knew the answer / solved the question

Attempted

Guessed between two/ eliminated two Guessed between three/ eliminated one Blind Guess Knew the answer / solved the question

Attempted

7/56

1/25/2016

Vision IAS Prelims

With reference to Union budget, which of the following is/are covered under the revenue receipts? 1.  National Saving certificates 2.  Income from sale of Treasury bills 3.  Grants­in­aid from foreign countries

Medium Factual(F) Detailed Analysis (question_analysis.php? test_id=1763&package_id=1276&compare_with=3&qid=22904&et1=1&et2=9)

Incorrect  :­0.67 Marks  Time:43 Sec.

Select the correct answer using the code given below. (a) 1 and 2 only (b) 2 and 3 only (c) 1 and 3 only (d) 3 only

Savings Accounts, National Savings Certificates, etc.) come under the Capital receipts. Income from sale of Treasury bills is a part of government borrowings which comes under capital receipts. Grants­in­aid from foreign countries and international organisations come under the non­tax revenue of revenue receipts. Hence D is the correct answer.

Attempted

Guessed between two/ eliminated two Guessed between three/ eliminated one Blind Guess Knew the answer / solved the question

A receipt is a revenue receipt, if it satisfies the following two essential conditions:

10

(i) The receipt must not create a liability for the government. For example, taxes levied by the government are revenue receipts as they do not create any liability. However, any amount, borrowed by the government, is not a revenue receipt as it causes an increase in Satisfies both the Conditions the liability in terms of repayment of borrowings. (ii) The receipt must not cause decrease in the assets. For example, a receipt from sale of shares of a public enterprise is not a revenue receipt as it leads to a reduction in assets of the government.

With reference to the Quit India Movement, consider the following statements: 1.  Failure of Cripps Mission 2.  Popular discontent 3.  Rejection of Gandhi's Eleven­ point demand by Lord Irwin 4.  Allied reverses and British withdrawal from South­East Asia and Burma

b Medium Factual(F) Detailed Analysis (question_analysis.php? test_id=1763&package_id=1276&compare_with=3&qid=22903&et1=1&et2=9)

Which of the above was/were the reasons to launch Quit India Movement? (a) 1 and 2 only (b) 1, 2 and 4 only http://visionias.in/student_open/pt/pta_form.php?test_id=1763&package_id=1276&compare_with=3

b Correct :2 Marks  Time:1 Sec.

Gandhi’s Eleven­point demands were not put during Quit India Movement. In January 1930, Gandhiji issued an eleven­point ultimatum to Lord Irwin, combining some general and specific demands. With no positive response forthcoming from the Government on Gandhi's Eleven­point demands, the Congress Working Committee invested Gandhi with full powers to launch the Civil Disobedience Movement (CDM) at a time and place of his choice.

Attempted

Guessed between two/ eliminated two Guessed between three/ eliminated one Blind Guess Knew the answer / solved the question

8/56

1/25/2016

Vision IAS Prelims

(c) 3 and 4 only 11

Reasons for launching Quit India Movement:

(d) 1, 2, 3 and 4

Failure of Cripps Mission in April 1942 made clear that Britain was determined to continue India's unwilling partnership in the War effort. Popular discontent, a product of rising prices and war­time shortages was mounting The news of Allied reverses and British withdrawals from South­ East Asia and Burma affirmed an imminent British collapse

For income accounting, the domestic territory of a country includes which of the following? 1.  Territory within the political frontiers 2.  Oil and natural gas rigs operated by the country in international waters 3.  Embassies of other countries located on its own territory

b Medium Analytical(A) Detailed Analysis (question_analysis.php? test_id=1763&package_id=1276&compare_with=3&qid=22902&et1=1&et2=9)

a  Incorrect  :­0.67 Marks  Time:74 Sec.

Select the correct answer using the code given below. 12

(a) 1 only (b) 1 and 2 only (c) 2 and 3 only (d) 1 and 3 only

The domestic territory of a country includes: 1.  Territory living within political frontiers, including its territorial waters. 2.  Ships and aircraft operated by the country between political frontiers of two or more countries. 3.  Oil and natural gas rigs, fishing vessels and floating platforms operated by the country in international waters. 4.  Embassies, military establishments and consulates of the country located in other countries of the world.

Attempted

Guessed between two/ eliminated two Guessed between three/ eliminated one Blind Guess Knew the answer / solved the question

It does not include the embassies of other countries and international organisations located on its own territory.

Which of the following type of unemployment occurs mainly due to changes in technology and preferences? (a) Cyclical Unemployment (b) Structural Unemployment

b Medium Analytical(A) Detailed Analysis (question_analysis.php? test_id=1763&package_id=1276&compare_with=3&qid=22901&et1=1&et2=9)

(c) Frictional Unemployment (d) Seasonal Unemployment  http://visionias.in/student_open/pt/pta_form.php?test_id=1763&package_id=1276&compare_with=3

b Correct :2 Marks  Time:24 Sec.

Structural Unemployment: This kind of unemployment happens when the structure of an industry changes. For example, as the

Attempted

Guessed between two/ eliminated two Guessed between three/ eliminated one 9/56

1/25/2016

Vision IAS Prelims

country is tending to move from use of bicycles to motorbikes and cars, the demand for labour in the cycle industry has continuously fallen in the country. Changes in technology and changes in tastes are two big reasons for the occurring of structural unemployment in the economy.  One of the reasons why 12th plan focuses on skill development is to address the problem of structural unemployment in the country. Cyclical unemployment: This kind of unemployment occurs when all those who want to work cannot be employed because there is not enough demand in the market for their work. Frictional Unemployment: This kind of unemployment occurs when a person leaves/loses a job and starts looking for another one. This search for a job may take a considerable amount of time resulting in frictional unemployment. Seasonal Unemployment: This kind of unemployment is expected to occur at certain parts of the year. 

13

The area known as 'Farzad B' sometimes appears in the news is in the context of

a

http://visionias.in/student_open/pt/pta_form.php?test_id=1763&package_id=1276&compare_with=3

not attempted

The Farzad B gas field is an Iranian natural gas field

Blind Guess Knew the answer / solved the question

Unattempted

10/56

1/25/2016

Vision IAS Prelims

(a) It is a Natural Gas field in the Persian Gulf. (b) It is a zone of conflict between Israel and Palestine. 14

Medium Factual(F) Detailed Analysis (question_analysis.php? test_id=1763&package_id=1276&compare_with=3&qid=22900&et1=1&et2=9)

that was discovered in 2012. The total in place reserves of the Farzad B gas field are around 21.7 trillion cubic feet. Government of India wants to participate in the extraction of gas in the Farzad­B field and has requested the Iranian government to complete the procedures for the same. 

(c) It is an oil field located in Iraq which was recently captured by ISIS. (d) It is the region of Tunisia where Arab Spring started.

In the Forum for India Pacific Islands Cooperation (FIPIC), an initiative of India and 14 Pacific Island nations, which of the following is/are not a participant/ participants?

15

1.  2.  3.  4.  5. 

Fiji Papua New Guinea Seychelles Comoros Solomon Islands

c

not attempted

Very difficult Factual(F) Detailed Analysis (question_analysis.php? test_id=1763&package_id=1276&compare_with=3&qid=22899&et1=1&et2=9)

Select the correct answer using the code given below. (a) 1 and 2 only (b) 2, 3 and 5 only (c) 3 and 4 only

FIPIC includes 14 of the island countries ­ Cook Islands, Fiji, Kiribati, Marshall Islands, Micronesia, Nauru, Niue, Palau, Papua New Guinea, Samoa, Solomon Islands, Tonga, Tuvalu, and Vanuatu. Madagascar (the world's fourth largest island), Bahrain, Comoros, Maldives, Mauritius, Seychelles and Sri Lanka are island nations of Indian ocean. Hence C is the correct answer.

Absolutely no idea / Could not comprehend the question Not sure of the right answer Inability to recall technique/method Fatigue: not able to concentrate Guessed incorrectly

Unattempted

Absolutely no idea / Could not comprehend the question Not sure of the right answer Inability to recall technique/method Fatigue: not able to concentrate Guessed incorrectly

(d) 4 and 5 only

Consider the following pairs: Region often

 

Country

1. Raqqa     

:

Syria

2. Mosul   

:

Iraq

3. Gwoza   

:

Nigeria

in news

c Difficult Factual(F) Detailed Analysis (question_analysis.php? test_id=1763&package_id=1276&compare_with=3&qid=22898&et1=1&et2=9)

16

http://visionias.in/student_open/pt/pta_form.php?test_id=1763&package_id=1276&compare_with=3

c Correct :2 Marks  Time:12 Sec.

Al­Raqqah, also called Raqqa, is a city in Syria located on the north bank of the Euphrates River. During the Syrian Civil War, in 2013, the city was captured by the Islamic State of Iraq and the Levant, which made it its headquarters in Syria. Mosul, is a city of northern Iraq, occupied since 10 June 2014 by the Islamic State salafi­jihadi militant group and therefore a major city in its self­ declared caliphate. Gwoza is a Local

Attempted

Guessed between two/ eliminated two Guessed between three/ eliminated one Blind Guess Knew the answer / solved the question

11/56

1/25/2016

Vision IAS Prelims

Government Area of Borno State, Nigeria. Gwoza has been called "a notorious hide out for the Boko Haram insurgents. In March 2015, Boko Haram, announced its allegiance to the Islamic State.

Which of the above pairs is/are correctly matched? (a) 1 only (b) 2 and 3 only (c) 1, 2 and 3 (d) None

In the context of modern scientific research, consider the following statements about 'LISA Pathfinder', a spacecraft which was recently in news: 1.  Its objective is to detect the dark matter. 2.  It is placed in space because Earth’s gravity may hamper the test results.

b Difficult Factual(F) Detailed Analysis (question_analysis.php? test_id=1763&package_id=1276&compare_with=3&qid=22897&et1=1&et2=9)

c  Incorrect  :­0.67 Marks  Time:46 Sec.

LISA Pathfinder is paving the way for future missions by testing in flight the very concept of gravitational wave detection: it will put two test masses in a near­ perfect gravitational free­ fall and control and measure their motion with unprecedented accuracy. Hence Statement 1 is wrong. It's objective is to detect the gravitational waves and not the dark matter. LISA Pathfinder is an ESA mission, which also carries a NASA payload. Only feasible in space LISA pathfinder is a pioneer mission. Not only are these technologies new, they cannot be properly verified on the ground. This is because Earth's gravity and environment would hamper the test rsults. Thus only Statement 2 is correct.

a Correct :2 Marks  Time:76 Sec.

The Comprehensive Nuclear­Test­Ban Treaty (CTBT) is a multilateral treaty by which states agree to ban all nuclear explosions in all environments, for military or civilian purposes. It was adopted by the United Nations General Assembly on 10 September 1996 but has not entered into force.

Which of the statements given above is/are correct? (a) 1 only (b) 2 only 17

(c) Both 1 and 2 (d) Neither 1 nor 2

With reference to the Comprehensive Nuclear Test Ban Treaty (CTBT), consider the following statements. 1.  It seeks to ban nuclear explosions for both military and civilian purposes. 2.  India has signed the treaty but not ratified it. 18

a Medium Factual(F) Detailed Analysis (question_analysis.php? test_id=1763&package_id=1276&compare_with=3&qid=22896&et1=1&et2=9)

Which of the statements given above is/are correct? (a) 1 only

http://visionias.in/student_open/pt/pta_form.php?test_id=1763&package_id=1276&compare_with=3

Attempted

Guessed between two/ eliminated two Guessed between three/ eliminated one Blind Guess Knew the answer / solved the question

Attempted

Guessed between two/ eliminated two Guessed between three/ eliminated one Blind Guess Knew the answer / solved 12/56

1/25/2016

Vision IAS Prelims

(b) 2 only

India has neither signed the treaty nor ratified it. Hence only statement 1 is correct.

(c) Both 1 and 2 (d) Neither 1 nor 2

Which of the following countries of Latin America open out to both Atlantic and Pacific Oceans? 1.  2.  3.  4.  19

Panama  Ecuador Venezuela Colombia

c

not attempted

Difficult Factual(F) Detailed Analysis (question_analysis.php? test_id=1763&package_id=1276&compare_with=3&qid=22895&et1=1&et2=9)

Select the correct answer using the code given below. (a) 1 only (b) 2 and 3 only (c) 1 and 4 only

Of the given countries, only Panama and Colombia open out to both Atlantic and Pacific oceans. Ecuador open out to only Pacific Ocean Venezuela open out to only Atlantic Ocean. Other countries which open to both Atlantic and Pacific ocean: Canada, USA, Mexico, Gautemala, Honduras, Nicaragua, Costa Rica and Chille. 

(d) 1, 3 and 4 only

With reference to 'Food Safety and Standards Authority of India' (FSSAI), consider the following statements:

20

1.  It is a nodal statutory agency responsible for regulation and supervision of food safety in India. 2.  It operates under the aegis of Union Ministry of Food Processing Industries. 3.  It is involved in the procurement of food products under price support scheme.

a

not attempted

Medium Analytical(A) Detailed Analysis (question_analysis.php? test_id=1763&package_id=1276&compare_with=3&qid=22894&et1=1&et2=9)

Which of the statements given above is/are correct? (a) 1 only (b) 1 and 2 only (c) 2 and 3 only (d) 1,2 and 3

Satyagraha Sabha was set up by Gandhiji to

b

http://visionias.in/student_open/pt/pta_form.php?test_id=1763&package_id=1276&compare_with=3

not attempted

Statement 1 is correct: The Food Safety and Standards Authority of India (FSSAI) is a nodal statutory agency responsible for protecting and promoting public health in India through the regulation and supervision of food safety. Statement 2 is incorrect: FSSAI was established under the Food Safety and Standards Act, 2006 and operates under aegis of Union Ministry of Health & Family Welfare. Statement 3 is incorrect: Food Corporation of India is involved in the procurement of food products under price support scheme. It was in new regularly due to Maggi controversy.

To protest against the Rowlatt Act (this act

the question

Unattempted

Absolutely no idea / Could not comprehend the question Not sure of the right answer Inability to recall technique/method Fatigue: not able to concentrate Guessed incorrectly

Unattempted

Absolutely no idea / Could not comprehend the question Not sure of the right answer Inability to recall technique/method Fatigue: not able to concentrate Guessed incorrectly

Unattempted

13/56

1/25/2016

Vision IAS Prelims

(a) create nation­wide campaign during Quit India movement (b) create nation­wide propaganda against Rowlatt Act

Medium Factual(F) Detailed Analysis (question_analysis.php? test_id=1763&package_id=1276&compare_with=3&qid=22893&et1=1&et2=9)

authorized the government to imprison for a maximum period of two years, without trial, any person suspected of terrorism), in February 1919 Gandhiji founded Satyagraha Sabha whose members took a pledge to disobey the Act and thus to court arrest and imprisonment. He organized a mass protest at all India level. By March 23, 1919, the volunteers started courting arrests. The three organizations viz, the Home Rule league, Muslim league and the Satyagraha Sabha along with some other small organizations coordinated and organized the biggest Satyagraha ever.   

(c) promote constructive work in villages for upliftment of Harijans and to popularize the use of Khadi (d) None of the above  21

What was the reason for drafting Nehru Report by the nationalists? (a) to raise demands to boycott Simon agitations.

22

(b) it came as a reaction to the defeat of the Public Safety Bill in 1928 in the Central Legislative Assembly.

d

not attempted

Easy Factual(F) Detailed Analysis (question_analysis.php? test_id=1763&package_id=1276&compare_with=3&qid=22892&et1=1&et2=9)

(c) it was brought to popularize the adoption of 'Purna Swaraj' at the Lahore Congress Session. 

As an answer to Lord Birkenhead's challenge given to Indian politicians to produce an agreed constitution, an All Parties Conference met in Feb.1928 and appointed a sub­committee under the chairmanship of Motilal Nehru to draft a Constitution, and thus Nehru Report came up .

(d) None of the above  

Consider the following: 1.  2.  3.  4. 

Sankirtana Bihu dance Kalbelia dance Ramlila

c Difficult Factual(F) Detailed Analysis (question_analysis.php? test_id=1763&package_id=1276&compare_with=3&qid=22891&et1=1&et2=9)

Which of the above from India are part of UNESCO's Intangible Cultural Heritage list?  (a) 1, 2 and 4 only (b) 2 and 3 only (c) 1, 3 and 4 only http://visionias.in/student_open/pt/pta_form.php?test_id=1763&package_id=1276&compare_with=3

not attempted

Except Bihu dance, rest all forms part of UNESCO's Intangible Cultural Heritage list. UNESCO's Intangible Cultural Heritage list: 1.  Traditional brass and copper craft of utensil making among the Thatheras of Jandiala Guru, Punjab 2.  Sankirtana, ritual singing, drumming and dancing of Manipur

Absolutely no idea / Could not comprehend the question Not sure of the right answer Inability to recall technique/method Fatigue: not able to concentrate Guessed incorrectly

Unattempted

Absolutely no idea / Could not comprehend the question Not sure of the right answer Inability to recall technique/method Fatigue: not able to concentrate Guessed incorrectly

Unattempted

Absolutely no idea / Could not comprehend the question Not sure of the right answer Inability to recall technique/method Fatigue: not able to concentrate Guessed 14/56

1/25/2016

Vision IAS Prelims

(d) 1, 2, 3 and 4

3.  Buddhist chanting of Ladakh: recitation of sacred Buddhist texts in the trans­Himalayan Ladakh region, Jammu and Kashmir, India 4.  Chhau dance a genre of Indian tribal martial dance which is popular in the Indian states of Odisha, Jharkhand and West Bengal. 5.  Kalbelia folk songs and dances of Rajasthan 6.  Mudiyettu, ritual theatre and dance drama of Kerala 7.  Novruz, Nowrouz, Nooruz, Navruz, Nauroz, Nevruz 8.  Ramman, religious festival and ritual theatre of the Garhwal Himalayas, India 9.  Kutiyattam, Sanskrit theatre 10.  Ramlila, the traditional performance of the Ramayana 11.  Tradition of Vedic chanting

23

Which of the following inscriptions was installed in the westernmost part of Ashoka's territory? (a) Kandahar pillar edicts (b) Dhauli major rock edicts 24

a Easy Analytical(A) Detailed Analysis (question_analysis.php? test_id=1763&package_id=1276&compare_with=3&qid=22890&et1=1&et2=9)

a Correct :2 Marks  Time:39 Sec.

Kandahar pillar edicts­ Afghanistan Dhauli major rock edicts­ Bhubaneshwar, Orissa( easternmost) Girnar major rock edicts­ Junagarh, Gujarat Lauriya Araraj pillar edicts­ Champaran, Bihar By recalling the map also this question can be answered.

d  Incorrect  :­0.67 Marks  Time:19 Sec.

Statement 1 is correct: Jainism mainly aims at the attainment of freedom from worldly bonds. It believes that liberation can be obtained through full knowledge and action and it is not necessary to use any ritual for acquiring such liberation. Full knowledge, action and liberation are considered to be the three gems or 'ratnas' of Jainism.

(c) Girnar major rock edicts (d) Lauriya Araraj pillar edicts 

Which of the following statements is/are applicable to Jain doctrine? 1.  It believes that liberation can be obtained through full knowledge and action. 2.  It does not recognize the existence of the Gods. 3.  It condemned the Varna system.

25

a Difficult Factual(F) Detailed Analysis (question_analysis.php? test_id=1763&package_id=1276&compare_with=3&qid=22889&et1=1&et2=9)

Select the correct answer using the code given below. (a) 1 only

http://visionias.in/student_open/pt/pta_form.php?test_id=1763&package_id=1276&compare_with=3

incorrectly

Attempted

Guessed between two/ eliminated two Guessed between three/ eliminated one Blind Guess Knew the answer / solved the question

Attempted

Guessed between two/ eliminated two Guessed between three/ eliminated one Blind Guess Knew the answer / solved 15/56

1/25/2016

Vision IAS Prelims

(b) 2 and 3 only (c) 1 and 3 only (d) 1, 2 and 3

With reference to the Cabinet Mission sent to India by the British Government, consider the following statements: 1.  It upheld the unity of India and rejected the demand for separate Pakistan. 2.  It recommended to set up a directly elected Constituent Assembly to frame a constitution for the Indian Union. 3.  It provided for a strong centre dealing with the subjects of Foreign Affairs, Defence and Communications .

a

not attempted

Medium Factual(F) Detailed Analysis (question_analysis.php? test_id=1763&package_id=1276&compare_with=3&qid=22888&et1=1&et2=9)

Which of the statements given above is/are correct?  (a) 1 only 26

(b) 2 and 3 only (c) 1 and 3 only (d) None

Which of the following pairs is/are correctly matched? Temple

 

Architectural Style

b Difficult Factual(F) Detailed Analysis (question_analysis.php? test_id=1763&package_id=1276&compare_with=3&qid=22887&et1=1&et2=9)

http://visionias.in/student_open/pt/pta_form.php?test_id=1763&package_id=1276&compare_with=3

b Correct :2 Marks  Time:43 Sec.

Statement 2 is incorrect: It recognized the existence of gods but placed them lower than the jina. Statement 3 is incorrect: It did not condemn the Varna system unlike Buddhism. Hence A is the correct answer.

the question

Statement 1 is correct: The Cabinet Mission Plan, 1946 most importantly it rejected the demand of Muslim league for Pakistan and recommended for a Union of India. Statement 2 is incorrect: The Cabinet Mission Plan even though gave the right to draft constitution , but the Constituent Assembly was to be elected indirectly. Statement 3 is incorrect: The Plan greatly restricted the powers of the Centre by allocating to it only three subjects i.e. Foreign Affairs , Defence and Communications. The Centre was deprived of control over such important subjects needing uniformity viz. Currency and Coinage, Customs and Tariffs, Weights and Measures, Planning and Development and Inter­State Commerce. Generally, these subjects are assigned to the Centre in federations in the interest of integrated economic development of the country.

Unattempted

The Kanchi Kailasanathar temple is the oldest structure in Kanchipuram Located in Tamil Nadu, India, it is a Hindu temple in the Dravidian architectural style.

Absolutely no idea / Could not comprehend the question Not sure of the right answer Inability to recall technique/method Fatigue: not able to concentrate Guessed incorrectly

Attempted

Guessed between two/ eliminated two Guessed between three/ 16/56

1/25/2016

27

Vision IAS Prelims

1. Kailasanath of Kanchipuram     

:

Chalukyan

2. Hoysaleswara

:

Vesara

3. Brihadeshwara

:

Dravida

Hoysala temples are of hybrid or vesara style as their unique architecture seems neither completely dravida nor nagara, but somewhere in between.

eliminated one Blind Guess Knew the answer / solved the question

Brihadeshwara temple is an architectural example showcasing the pure form of the Dravida type of temple architecture and representative of the Chola Empire ideology and the Tamil civilisation in Southern India.

Select the correct answer using the code given below.   (a) 1 and 2 only

Hence B is the correct answer.

(b) 2 and 3 only (c) 3 only (d) None

Madan Mohan Malviya was associated with which of the following: 1.  2.  3.  4. 

Banaras Hindu University  Hindustan Times Hindu Mahasabha Indian National Congress

d

not attempted

Medium Factual(F) Detailed Analysis (question_analysis.php? test_id=1763&package_id=1276&compare_with=3&qid=22886&et1=1&et2=9)

Select the correct answer using the code given below. (a) 1 only (b) 1, 3 and 4 only 28

(c) 2, 3 and 4 only (d) 1, 2, 3 and 4

With reference to India's culture and tradition, what is 'Pavakoothu'? (a) It is a religious offering given at Sabarimala temple.

c Difficult Factual(F) Detailed Analysis (question_analysis.php?

http://visionias.in/student_open/pt/pta_form.php?test_id=1763&package_id=1276&compare_with=3

not attempted

Malaviya is most remembered as the founder of Banaras Hindu University. He was the President of the Indian National Congress on two occasions (1909, 1918). He left Congress in 1934. He was a member of the Hindu Mahasabha. He was a president of the special session of Hindu Mahasabha in Gaya in 1922 and in Kashi in 1923. He was the Chairman of Hindustan Times from 1924 to 1946. His efforts resulted in the launch of its Hindi edition named Hindustan Dainik in 1936. He was also awarded Bharat Ratna recently. Hence D is the correct answer.

In Kerala, the traditional glove puppet play is called Pavakoothu. It came into existence during the 18th century due to the

Unattempted

Absolutely no idea / Could not comprehend the question Not sure of the right answer Inability to recall technique/method Fatigue: not able to concentrate Guessed incorrectly

Unattempted

Absolutely no idea / Could not 17/56

1/25/2016

Vision IAS Prelims

(b) It is a religious festival celebrated on a grand scale in some parts of South Andhra Pradesh.

test_id=1763&package_id=1276&compare_with=3&qid=22885&et1=1&et2=9)

influence of Kathakali, the famous classical dance­ drama of Kerala, on puppet performances.The theme for Glove puppet plays in Kerala is based on the episodes from either the Ramayana or the Mahabharata. In Pavakoothu, the height of a puppet varies from one foot to two feet. The head and the arms are carved of wood and joined together with thick cloth, cut and stitched into a small bag. The musical instruments used during the performance are Chenda, Chengiloa, Ilathalam and Shankhathe conch.

(c) It is an ancient form of puppet play performed in Kerala.

29

(d) It is an ancient form of martial art practised in some parts of South India.

Which of the following statements is/are correct regarding Brahmo Samaj? 1.  It opposed priesthood. 2.  It opposed the worship of idols, images or statues. 3.  It opposed the doctrine of Karma and transmigration of soul. 30

b Medium Factual(F) Detailed Analysis (question_analysis.php? test_id=1763&package_id=1276&compare_with=3&qid=22884&et1=1&et2=9)

d  Incorrect  :­0.67 Marks  Time:43 Sec.

Brahmo Samaj was founded by Raja Rammohan Roy. He criticised idolatory and re­ interpreted Hindu doctrines. There was no place for preisthood in the Samaj building. Brahmo Samaj started a campaign for the abolition of Sati, condemned polygamy, denounced casteism. Brahmo Samaj took no definite stand on the doctrine of Karma and transmigration of soul and left it to individual Brahmos to believe either ways. 

not attempted

Constructive plate boundaries represent the trailing ends of divergent plates and shallow focus earthquakes are associated with divergent plates. So, the first statement is correct. Mid­oceanic ridges are associated with divergent plates. These are regions of shallow focus earthquakes and not deep

Select the correct answer using the code given below. (a) 1 only (b) 1 and 2 only (c) 3 only (d) 1 ,2 and 3

With reference to the depth of the earthquakes, consider the following statements: 1.  Shallow focus earthquakes are caused along the constructive plate boundaries. 2.  Deep focus earthquakes are caused along the mid­oceanic ridges. 31

a Medium Analytical(A) Detailed Analysis (question_analysis.php? test_id=1763&package_id=1276&compare_with=3&qid=22883&et1=1&et2=9)

Which of the statements given above is/are correct? (a) 1 only

http://visionias.in/student_open/pt/pta_form.php?test_id=1763&package_id=1276&compare_with=3

comprehend the question Not sure of the right answer Inability to recall technique/method Fatigue: not able to concentrate Guessed incorrectly

Attempted

Guessed between two/ eliminated two Guessed between three/ eliminated one Blind Guess Knew the answer / solved the question

Unattempted

Absolutely no idea / Could not comprehend the question Not sure of the right answer Inability to recall technique/method Fatigue: not able to concentrate 18/56

1/25/2016

Vision IAS Prelims

(b) 2 only

focus earthquakes. Therefore, this statement is not correct.

(c) Both 1 and 2

Guessed incorrectly

(d) Neither 1 nor 2

Which of the following categories is/are included under Protected Area network in India? 1.  2.  3.  4. 

Wildlife Sanctuary Conservation Reserve Community Reserve National Park

d

not attempted

Easy Factual(F) Detailed Analysis (question_analysis.php? test_id=1763&package_id=1276&compare_with=3&qid=22882&et1=1&et2=9)

All are included under Protected Area network in India. As of 2014 there are 690 PAs (102 National Parks, 527 Wildlife Sanctuaries, 57 Conservation Reserves and 4 Community Reserves) covering 1,66,851 km or 5.07% of the country's geographical area. Biosphere Reserves are not the part of Protected area network.

Select the correct answer using the code given below. 32

(a) 1 and 4 only (b) 2 and 3 only (c) 1, 2 and 4 only (d) 1, 2, 3 and 4

Which of the following statements is/are correct about Mangroves? 1.  It can grow along the intertidal zones in tropical and subtropical coastal regions. 2.  East coast of India has more Mangrove cover than the West coast.

c Medium Analytical(A) Detailed Analysis (question_analysis.php? test_id=1763&package_id=1276&compare_with=3&qid=22881&et1=1&et2=9)

Select the correct answer using the code given below.  (a) 1 only (b) 2 only (c) Both 1 and 2 33

Protected areas or conservation areas are locations which receive protection because of their recognized natural, ecological and/or cultural values. 

(d) Neither 1 nor 2

http://visionias.in/student_open/pt/pta_form.php?test_id=1763&package_id=1276&compare_with=3

c Correct :2 Marks  Time:26 Sec.

Mangrove plants occupy shallow water and intertidal zones in tropical and subtropical coastal regions, usually where protected from direct wave action, and thus characterized by muddy or fine sediment substrata. Salient features of mangroves: 1.  Mangroves are practically evergreen with thick leathery leaves designed to minimize transpiration. 2.  Root system is adapted to the peculiar conditions found in the mangrove forests. 3.  Viviparous germination where the seed germinates while still on the tree and falls down in the germinating condition with a long radicle­ characteristic of Rhizophoraceae.

Unattempted

Absolutely no idea / Could not comprehend the question Not sure of the right answer Inability to recall technique/method Fatigue: not able to concentrate Guessed incorrectly

Attempted

Guessed between two/ eliminated two Guessed between three/ eliminated one Blind Guess Knew the answer / solved the question

19/56

1/25/2016

Vision IAS Prelims

About 59% of this cover is on the East coast, along the Bay of Bengal, 28% on the West coast, bordering the Arabian Sea and 13% on the Andaman and Nicobar Islands.

Consider the following statements regarding 'Raptor MoU': 1.  It deals with Conservation of Migratory Birds of Prey in Africa and Eurasia. 2.  It is an agreement under Bonn Convention. 3.  It is legally binding on the signatory parties.

b Medium Factual(F) Detailed Analysis (question_analysis.php? test_id=1763&package_id=1276&compare_with=3&qid=22880&et1=1&et2=9)

d  Incorrect  :­0.67 Marks  Time:32 Sec.

India has signed Memoranda of Understanding (MoU) on the Conservation of Migratory Birds of Prey in Africa and Eurasia, also called the 'Raptor MoU. Raptor MOU is also in conformity with the provisions of the existing Wild Life (Protection) Act, 1972, wherein the birds have been accorded protection. Raptor MoU is not legally binding and comes under the Convention on Conservation of Migratory Species (CMS) or Bonn Convention under the auspices of the United Nations Environment Programme (UNEP), which aims to conserve migratory species throughout their range.

c  Incorrect  :­0.67 Marks  Time:12 Sec.

A boundary transition zone between different, but adjoining ecosystems at any scale is an ecotone. The range of environmental conditions found in ecotones makes them areas of high biodiversity and often larger population densities than communities on either side.Hence only statement 2 is correct and option b is the correct answer. Zone of interaction of many communities with the abiotic physical components of its environment is called Ecosystem.

Which of the statements given above is/are correct? 34

(a) 1 only  (b) 1 and 2 only (c) 2 and 3 only (d) 1, 2 and 3

Consider the following statements about 'Ecotone': 1.  Ecotone refers to the population of different kinds of organisms living together and sharing the same habitat. 2.  Ecotones are the areas of high biodiversity. 35

b Easy Analytical(A) Detailed Analysis (question_analysis.php? test_id=1763&package_id=1276&compare_with=3&qid=22879&et1=1&et2=9)

Which of the statements given above is/are correct? (a) 1 only (b) 2 only (c) Both 1 and 2 (d) Neither 1 nor 2

http://visionias.in/student_open/pt/pta_form.php?test_id=1763&package_id=1276&compare_with=3

Attempted

Guessed between two/ eliminated two Guessed between three/ eliminated one Blind Guess Knew the answer / solved the question

Attempted

Guessed between two/ eliminated two Guessed between three/ eliminated one Blind Guess Knew the answer / solved the question

20/56

1/25/2016

Vision IAS Prelims

From the point of view of evolution of living organisms, which one of the following is the correct sequence of evolution? (a) Cat fish ­ Tortoise ­ Otter ­ Kiwi (b) Cat fish ­ Kiwi ­ Tortoise ­ Otter

d

not attempted

Difficult Analytical(A) Detailed Analysis (question_analysis.php? test_id=1763&package_id=1276&compare_with=3&qid=22878&et1=1&et2=9)

(c) Tortoise ­ Cat fish ­ Otter ­ Kiwi 36

(d) Cat fish ­ Tortoise ­ Kiwi ­ Otter

You are walking along the McMohan Line from west to east 1.  If you jump back to the Indian side, you will find yourself in Sikkim. 2.  The latitude passing from your location will cut through Nagaland and Rajasthan. 3.  At the end of your journey along the line you may end up in Myanmar. 37

d

not attempted

Medium Analytical(A) Detailed Analysis (question_analysis.php? test_id=1763&package_id=1276&compare_with=3&qid=22877&et1=1&et2=9)

(a) 1 only (b) 1 and 2 only (c) 2 and 3 only (d) 3 only

1.  2.  3.  4. 

Chlorofluorocarbons Hydrofluorocarbons Hydrochlorofluorocarbons Halons

The McMohan Line runs from the tri­junction of India, Bhutan and China to the tri­junction of India, China and Myanmar. It's eastern sector of the international border between India and China along the state of Arunachal Pradesh. Statement 1 is incorrect: If you jump back to the Indian side, you will find yourself in Arunachal Pradesh. Statement 2 is incorrect: The latitude passing from your location will cut through Rajasthan but not Nagaland. Statement 3 is correct: At the end of the line there is tri­junction of India, China and Myanmar.

Which of the of the statements given above is/are correct?

Consider the following:

While it is estimated that the Earth is 4.5 billion years old, life began to appear approximately 3.5 billion years ago. Evolution of vertebrates (those having a backbone) occurred in the following order: Fish, Amphibia, Reptiles, Birds and Mammals. Hence, the correct sequence is Cat fish, Tortoise, Kiwi, Otter.

d Medium Analytical(A) Detailed Analysis (question_analysis.php? test_id=1763&package_id=1276&compare_with=3&qid=22876&et1=1&et2=9)

Which of the above are responsible for depletion of the ozone layer? (a) 1 and 2 only (b) 1, 2 and 3 (c) 3 and 4 only (d) 1, 3 and 4

http://visionias.in/student_open/pt/pta_form.php?test_id=1763&package_id=1276&compare_with=3

d Correct :2 Marks  Time:13 Sec.

Chlorofluorocarbons (CFCs) are ozone depleting chemicals containing atoms of carbon, chlorine, and fluorine. They are used in the manufacture of aerosol sprays, blowing agents for foams and packing materials, as solvents, and as refrigerants. Hydrofluorocarbon (HFC), any of several organic compounds composed of hydrogen, fluorine, and carbon. HFCs are produced synthetically and

Unattempted

Absolutely no idea / Could not comprehend the question Not sure of the right answer Inability to recall technique/method Fatigue: not able to concentrate Guessed incorrectly

Unattempted

Absolutely no idea / Could not comprehend the question Not sure of the right answer Inability to recall technique/method Fatigue: not able to concentrate Guessed incorrectly

Attempted

Guessed between two/ eliminated two Guessed between three/ eliminated one Blind Guess Knew the answer / solved the question

21/56

1/25/2016

Vision IAS Prelims

are used primarily as refrigerants. Hydrochlorofluorocarbons (HCFCs) are a large group of compounds, whose structure is very close to that of Chlorofluorocarbons (CFCs), but including one or more hydrogen atoms. Under normal conditions, HCFCs are gases or liquids which evaporate easily. Although they do not have high ODP, they have a high GWP (Global Warming Potential) Halons are a group of chemical compounds containing bromine, chlorine, fluorine or carbon in its structure. Halons are very effective for extinguishing fires. Although halons do not present a direct hazard to people, they have a very high ozone­depleting potential (ODP) because they contain bromine.

38

With regard to convertibility of currency, consider the following statements: 1.  If the capital account is fully convertible, importers and exporters are allowed free conversion of rupee. 2.  Capital account of India is fully convertible. 3.  India is planning to achieve fully convertible current account in the next 5 years. 39

d Medium Analytical(A) Detailed Analysis (question_analysis.php? test_id=1763&package_id=1276&compare_with=3&qid=22875&et1=1&et2=9)

Which of the statements given above is/are correct? (a) 1 and 3 only (b) 1 and 2 only (c) 3 only (d) None

http://visionias.in/student_open/pt/pta_form.php?test_id=1763&package_id=1276&compare_with=3

d Correct :2 Marks  Time:63 Sec.

Statement 1 is incorrect. Importers and exporters are allowed free conversion of rupee in case of current account convertibility. Statement 2 is incorrect. India has partial Capital Account Convertibility (CAC). Capital Account Convertibility means that rupee can now be freely convertible into any foreign currencies for acquisition of assets like shares, properties and assets abroad. At present, there are limits on investment by foreign financial investors and also caps on FDI ceiling in most sectors. Statement 3 is incorrect. India's current account is today fully convertible (operationalized on August 19, 1994).

Attempted

Guessed between two/ eliminated two Guessed between three/ eliminated one Blind Guess Knew the answer / solved the question

22/56

1/25/2016

Vision IAS Prelims

With regard to taxation, consider the following statements: 1.  Progressive taxes reduce the incidence of taxes on people with lower incomes. 2.  In case of proportional taxation, the tax rate increases with income. 3.  Sales tax on food items is an example of regressive tax.

d

not attempted

Difficult Analytical(A) Detailed Analysis (question_analysis.php? test_id=1763&package_id=1276&compare_with=3&qid=22874&et1=1&et2=9)

Which of the statements given above is/are correct? (a) 1 only (b) 1 and 2 only (c) 2 and 3 only (d) 1 and 3 only 40

Which of the following components is/are used in calculating Human Development Index (HDI) by UNDP? 1.  Life expectancy at birth 2.  Mean years of schooling and expected years of schooling 3.  GNP per capita  41

b Medium Factual(F) Detailed Analysis (question_analysis.php? test_id=1763&package_id=1276&compare_with=3&qid=22873&et1=1&et2=9)

Select the correct answer using the code given below. (a) 1 only (b) 1 and 2 only

d  Incorrect  :­0.67 Marks  Time:10 Sec.

Statement 1 is correct. Progressive tax is a tax in which the tax rate increases as the tax base increases. Here the rate of taxation increases as the income increases. Progressive taxes reduce the incidence of taxes on people with lower incomes. Thus people with lower ability­to­pay, actually pay less. Statement 2 is incorrect. Proportional taxation is a tax where the rate of taxation is fixed. The amount of the tax is a fixed proportion of one's income. It is also called as flat tax. Statement 3 is correct. Sales tax on food items is an example of regressive tax. Sales taxes on food items tend to make up a higher percentage of a lower income consumer's overall budget. However it makes up a very low percentage of income in a higher income consumer’s budget. In this case, even though the tax may be uniform, lower income consumers are more affected by it because they are less able to afford it.

The Human Development Index (HDI) by UNDP uses the following components: Life expectancy at birth Mean of years of schooling for adults aged 25 years Expected years of schooling for children of school entering age GNI per capita (PPP$)

Unattempted

Absolutely no idea / Could not comprehend the question Not sure of the right answer Inability to recall technique/method Fatigue: not able to concentrate Guessed incorrectly

Attempted

Guessed between two/ eliminated two Guessed between three/ eliminated one Blind Guess Knew the answer / solved the question

(c) 2 and 3 only (d) 1, 2 and 3

http://visionias.in/student_open/pt/pta_form.php?test_id=1763&package_id=1276&compare_with=3

23/56

1/25/2016

Vision IAS Prelims

Which of the following measures is/are likely to reduce inflation in the economy? 1.  Remove levy obligations in respect of imported materials. 2.  Ban export of constrained materials. 3.  Increasing wages 42

b

not attempted

Medium Analytical(A) Detailed Analysis (question_analysis.php? test_id=1763&package_id=1276&compare_with=3&qid=22872&et1=1&et2=9)

Select the correct answer using the code given below. (a) 1 only (b) 1 and 2 only (c) 3 only (d) 1, 2 and 3

Which of the following negatively affects the exchange rate of a domestic currency? 1.  2.  3.  4. 

Increase in exports Increase in imports Lower interest rate Higher inflation rate

b Difficult Analytical(A) Detailed Analysis (question_analysis.php? test_id=1763&package_id=1276&compare_with=3&qid=22871&et1=1&et2=9)

b Correct :2 Marks  Time:69 Sec.

Select the correct answer using the code given below. (a) 1 and 4 only (b) 2, 3 and 4 only (c) 1, 2 and 3 only 43

(d) 2 and 3 only

Which of the following statements regarding the territorial limits of the legislative powers vested in the Centre and the States is/are

c Difficult

http://visionias.in/student_open/pt/pta_form.php?test_id=1763&package_id=1276&compare_with=3

d  Incorrect  :­0.67

Option 1 is correct. Removing levy obligations in respect of imported materials encourages more imports and reduces the demand for constrained materials. This reduces demand pull factors of inflation. Option 2 is correct. Ban export of constrained materials also provides for more domestic availability. Option 3 is incorrect. Rising wages provide for more money in the market and thus increases the inflation.

Increase in exports positively affects the exchange rate. Increase in exports of a country will lead to an increase in demand for the currency and thus its value rises.  Increase in imports will lead to higher demand for foreign currency and hence the demand for domestic currency reduces. This will negatively affect the exchange rate. Lower interest rate will attract less foreign investors to invest in the country and thus the demand for currency will decrease, resulting in depreciation in value of the currency. Higher inflation rate will make the country uncompetitive in the international market. The exports will fall resulting in decreased demand for the currency and hence lower value.

Unattempted

Absolutely no idea / Could not comprehend the question Not sure of the right answer Inability to recall technique/method Fatigue: not able to concentrate Guessed incorrectly

Attempted

Guessed between two/ eliminated two Guessed between three/ eliminated one Blind Guess Knew the answer / solved the question

Statement 1 is incorrect: The Parliament can make laws for the whole or any part of the 24/56

1/25/2016

Vision IAS Prelims

correct? 1.  The Constitution does not restrict the Parliament to make laws for the whole or any part of the territory of India. 2.  The Parliament alone can make laws applicable on Indian citizens and their property in any part of the world. 3.  The laws made by the state legislature are not applicable outside the state in any condition.

Factual(F) Detailed Analysis (question_analysis.php? test_id=1763&package_id=1276&compare_with=3&qid=22870&et1=1&et2=9)

Select the correct answer using the code given below. (a) 1 and 2 only (b) 1 and 3 only (c) 2 only (d) 1, 2 and 3

44

Marks  Time:17 Sec.

territory of India. However, the Constitution places certain restrictions on the plenary territorial jurisdiction of the Parliament i.e. the laws of the Parliament are not applicable in the following areas: (i) The President can make regulations for the peace, progress and good government of the four Union Territories—the Andaman and Nicobar Islands, Lakshadweep, Dadra and Nagar Haveli and Daman and Diu. A regulation so made has the same force and effect as an act of Parliament. It may also repeal or amend any act of Parliament in relation to these union territories.

Attempted

Guessed between two/ eliminated two Guessed between three/ eliminated one Blind Guess Knew the answer / solved the question

(ii) The Governor is empowered to direct that an act of Parliament does not apply to a scheduled area in the state or apply with specified modifications and exceptions. (iii) The Governor of Assam may likewise direct that an act of Parliament does not apply to a tribal area (autonomours district) in the state or apply with specified modifications and exceptions. The President enjoys the same power with respect to tribal areas (autonomous districts) in Meghalaya, Tripura and Mizoram. Statement 2 is correct: The Parliament alone can make 'extra­territorial legislation'. Thus, the laws made by the Parliament are also applicable to the Indian citizens and their property in any part of the world. Statement 3 is incorrect: Although the state legislature can make laws for the whole or any part of the state. However, the laws made by a state legislature are not applicable outside the state, except when there is a sufficient nexus between the state and the object.

http://visionias.in/student_open/pt/pta_form.php?test_id=1763&package_id=1276&compare_with=3

25/56

1/25/2016

Vision IAS Prelims

Consider the following statements: 1.  In case of Inter­State Water Disputes, Parliament may provide that neither the Supreme Court nor any other court is to exercise jurisdiction. 2.  Inter­State Water Disputes Act empowers the Parliament to set up an ad­hoc tribunal for adjudication of dispute.

c Medium Factual(F) Detailed Analysis (question_analysis.php? test_id=1763&package_id=1276&compare_with=3&qid=22869&et1=1&et2=9)

c Correct :2 Marks  Time:1 Sec.

Which of the statements given above is/are correct? 45

Article 262 of the constitution provides for the adjudication of the Inter­state water disputes. Parliament may by law provide for the adjudication of any dispute or complaint with respect to the use, distribution and control of waters of any inter­ state river. Parliament may also provide that neither the SC nor any other court is to exercise jurisdiction in respect of any such dispute or complaint.

(a) 1 only (b) 2 only

Attempted

Guessed between two/ eliminated two Guessed between three/ eliminated one Blind Guess Knew the answer / solved the question

The Inter­State Water Disputes (ISWD) Act empowers the central government to set up an ad hoc tribunal for the adjudication of a dispute between two or more states.

(c) Both 1 and 2  (d) Neither 1 nor 2

SC have jurisdiction to decide any dispute between states in connection with water supplies, if legal rights are concerned.

Which one of the following statements is not correct? (a) After the expiry of the term of Governor he may be reappointed in the same state or any other state.

d Easy Analytical(A) Detailed Analysis (question_analysis.php? test_id=1763&package_id=1276&compare_with=3&qid=22868&et1=1&et2=9)

d Correct :2 Marks  Time:50 Sec.

The Governor though appoints the chairman and members of State Public Service Commission but they are removed only by the President.

a  Incorrect  :­0.67 Marks  Time:50

Statement 1 is incorrect. The amount of oxygen required to break down a certain amount of organic matter is called the

(b) The oath of office to the Governor is administered by the Chief Justice of the concerned state high court. 46

(c) The Governor does not possess diplomatic, military or emergency powers like the President. (d) The Governor appoints and removes the chairman and members of the State Public Service Commission.

With regard to Biological Oxygen Demand (BOD) in an aquatic ecosystem, consider the following statements: 1.  BOD is the amount of oxygen

d Medium Analytical(A) Detailed Analysis (question_analysis.php?

http://visionias.in/student_open/pt/pta_form.php?test_id=1763&package_id=1276&compare_with=3

Attempted

Guessed between two/ eliminated two Guessed between three/ eliminated one Blind Guess Knew the answer / solved the question

Attempted

Guessed between two/ 26/56

1/25/2016

Vision IAS Prelims

required for the respiration of all aquatic organisms in an ecosystem. 2.  Higher the BOD, lower is the pollution level in the aquatic ecosystem.  47

test_id=1763&package_id=1276&compare_with=3&qid=22867&et1=1&et2=9)

Sec.

biological oxygen demand (BOD). Statement 2 is incorrect. Higher the BOD, higher is the pollution level. The amount of BOD in the water is an indicator of the level of pollution. If too much organic matter is added to the water all the available oxygen is used up. This causes fish and other forms of oxygen dependent aquatic life to die.

d

not attempted

A chemical reaction is usually accompanied by easily observed physical effects, such as the emission of heat and light, the formation of a precipitate, the evolution of gas, or a color change, however, absolute confirmation of a chemical change can only be validated by chemical analysis of the products. Change in state can occur with simple heating which is not always a chemical process. For example, conversion of ice into water. The changing of color of a substance is not necessarily an indicator of a chemical change. For example, changing the color of iron when being heated. Evolution of a gas can again happen with simple heating. Change in temperature doesn't require a chemical reaction.

Which of the statements given above is/are correct?  (a) 1 only (b) 2 only (c) Both 1 and 2 (d) Neither 1 nor 2

Which of the following definitely indicates a chemical change? 1.  2.  3.  4. 

Change in state Change in colour Evolution of a gas Change in temperature

Medium Analytical(A) Detailed Analysis (question_analysis.php? test_id=1763&package_id=1276&compare_with=3&qid=22866&et1=1&et2=9)

Select the correct answer using the code given below. (a) 1 and 3 only (b) 2 only (c) 1, 2, 3 and 4 (d) None 48

Consider the following properties of Carbon Nanofibers: 1.  Graphene layers are used in Carbon Nanofibres. 2.  Carbon dioxide (CO2) can be used to

d Very difficult Factual(F) Detailed Analysis (question_analysis.php? test_id=1763&package_id=1276&compare_with=3&qid=22865&et1=1&et2=9)

http://visionias.in/student_open/pt/pta_form.php?test_id=1763&package_id=1276&compare_with=3

not attempted

Carbon nanofibers (CNFs): It is cylindric nanostructures with graphene layers arranged as stacked cones, cups or plates structure. It has high

eliminated two Guessed between three/ eliminated one Blind Guess Knew the answer / solved the question

Unattempted

Absolutely no idea / Could not comprehend the question Not sure of the right answer Inability to recall technique/method Fatigue: not able to concentrate Guessed incorrectly

Unattempted

Absolutely no idea / Could not comprehend the 27/56

1/25/2016

Vision IAS Prelims

make Nanofibres 3.  They are used in electronic components and batteries.

tensile strength, less distortions with changes in temperature and high electromagnetic shielding. Graphene layers of CNFs wrapped into perfect cylinders are called carbon nanotubes. Applications of CFNs: They are already used in high­end applications such as electronic components and batteries. They are used as lightweight carbon composites materials in aircraft and car components. Recently, scientists in the US have found a way to take carbon dioxide (CO2) from the air and make carbon nanofibres. It could offer a cheaper way of making carbon nanofibres than existing costly methods which are too expensive for many applications. Therefore all statements are correct. Hence, D is the correct answer.

Which of the statements given above is/are correct? (a) 1 only (b) 1 and 3 only (c) 2 and 3 only (d) 1, 2 and 3 49

With reference to 'Project Loon' consider the following statements: 1.  It proposes to provide internet connectivity to remote areas using balloons floating in troposphere. 2.  It aims to tap unused TV spectrum to provide internet access. 3.  The project is being implemented by Google Inc.

d Difficult Factual(F) Detailed Analysis (question_analysis.php? test_id=1763&package_id=1276&compare_with=3&qid=22864&et1=1&et2=9)

Which of the statements given above is/are correct? (a) 1 and 3 only (b) 2 only 50

(c) 2 and 3 only (d) 3 only

http://visionias.in/student_open/pt/pta_form.php?test_id=1763&package_id=1276&compare_with=3

a  Incorrect  :­0.67 Marks  Time:28 Sec.

Statement 1 is incorrect: Project Loon is a research and development project being developed by Google X with the mission of providing Internet access to rural and remote areas. The project uses high­ altitude balloons placed in the stratosphere at an altitude of about 18 km (11 mi) to create an aerial wireless network with up to 4G­LTE speeds. Statement 2 is incorrect: By partnering with Telecommunications companies to share cellular spectrum Google is trying to enable people to connect to the balloon network directly from their phones and other LTE­ enabled devices. Microsoft

question Not sure of the right answer Inability to recall technique/method Fatigue: not able to concentrate Guessed incorrectly

Attempted

Guessed between two/ eliminated two Guessed between three/ eliminated one Blind Guess Knew the answer / solved the question

28/56

1/25/2016

Vision IAS Prelims

is planning to use unused TV spectrum to provide internet access in remote areas. Statement 3 is correct. Hence C is the correct answer. 

With reference to Bharat Stage (BS) emission standards, consider the following statements: 1.  "Bharat Stage" are emission standards in accordance with ‘American norms’ to regulate the output of air pollutants that vehicles emit. 2.  Currently, BS­IV norms are applicable in all cities of India. 3.  Government of India wants to directly implement BS­VI auto emission norms skipping BS­V.

a Difficult Factual(F) Detailed Analysis (question_analysis.php? test_id=1763&package_id=1276&compare_with=3&qid=22863&et1=1&et2=9)

d  Incorrect  :­0.67 Marks  Time:39 Sec.

Bharat stage emission standards are emission standards instituted by the Government of India to regulate the output of air pollutants from internal combustion engine equipment, including motor vehicles. The standards and the timeline for implementation are set by the Central Pollution Control Board under the Ministry of Environment & Forests and climate change. The standards, based on European regulation standard ‘Euro norms’were first introduced in 2000. BS­IV norms are currently applicable in 33 cities in which the required grade of fuel is available; the rest of India still conforms to BS­III standards. Four years from now, the government wants to leap directly to BS­VI auto emission norms from the existing BS­IV, skipping BS­V. Thus only statement 3 is correct. Hence D is the correct answer.

not attempted

A transistor is a miniature electronic component that can do two different jobs. It can work either as an amplifier or a switch.

Which of the statements given above is/are not correct?  51

(a) 1 and 2 only (b) 2 only (c) 2 and 3 only (d) 3 only

Transistor is used to

a

(a) amplify or switch electronic signals and electrical power

Medium Analytical(A) Detailed Analysis (question_analysis.php? test_id=1763&package_id=1276&compare_with=3&qid=22862&et1=1&et2=9)

(b) regulate the fluctuation of voltage (c) convert direct current to alternate current 52

(d) reduce the resistance in an electrical circuit 

http://visionias.in/student_open/pt/pta_form.php?test_id=1763&package_id=1276&compare_with=3

Attempted

Guessed between two/ eliminated two Guessed between three/ eliminated one Blind Guess Knew the answer / solved the question

Unattempted

Absolutely no idea / Could not comprehend the question Not sure of the right answer Inability to recall technique/method Fatigue: not able 29/56

1/25/2016

Vision IAS Prelims

to concentrate Guessed incorrectly

Rainbows are shaped as arcs and not straight line because: (a) the earth is round and not flat. (b) of the illusion in our eyes; in reality it is a straight line.

c

not attempted

Medium Analytical(A) Detailed Analysis (question_analysis.php? test_id=1763&package_id=1276&compare_with=3&qid=22861&et1=1&et2=9)

(c) there is a set of points in the sky where the sun, the cloud and eye form the same angle. (d) water droplets are conical in shape.

53

Consider the following states: 1.  Madhya Pradesh 2.  Tamil Nadu 3.  Odisha Which of the above states has/have 'Tropical Dry Evergreen Forest' cover?  54

b Medium Factual(F) Detailed Analysis (question_analysis.php? test_id=1763&package_id=1276&compare_with=3&qid=22860&et1=1&et2=9)

(a) 1 and 2 only (b) 2 only (c) 1 and 3 only (d) 2 and 3 only

not attempted

Light radiates out from the sun in all directions, and when it strikes water droplets and is refracted into our eyes at just the right angle (about 42 degrees) we perceive the light as a band of colors. When we face a cloud of water droplets with the sun behind us, there's a circle of points where the sun, the cloud and your eye form that angle. That circle is where we see the rainbow. The shape of the Earth doesn't come in to it because the Earth isn't involved in the process. It only involves sunlight, water, and your eye. The conical shape of droplets work as a Prism to form rainbow but are not the deciding factor of the shape of the rainbow. The semicircular arc is not an illusion. Hence 'C' is the correct answer.

Tropical dry evergreen forests occur in India only in Tamil Nadu, patches of Karnataka and Andhra Pradesh. These forests are found in the areas of relatively high temperature and small rainfall available only during particular period.

Unattempted

Absolutely no idea / Could not comprehend the question Not sure of the right answer Inability to recall technique/method Fatigue: not able to concentrate Guessed incorrectly

Unattempted

Absolutely no idea / Could not comprehend the question Not sure of the right answer Inability to recall technique/method Fatigue: not able to concentrate Guessed incorrectly

Attempted

http://visionias.in/student_open/pt/pta_form.php?test_id=1763&package_id=1276&compare_with=3

30/56

1/25/2016

Vision IAS Prelims

Which of the following pairs is incorrectly matched?

55

Drainage Pattern

 

Example

1. Dendritic

:

Rivers of Indo gangetic plains

2. Radial

:

Rivers originating from Amarkantak Hills

3. Annular

:

Rivers of the Chhotanagpur Plateau region

:

Rivers of western coastal plains draining into Arabian Sea

4. Parallel

c Very difficult Analytical(A) Detailed Analysis (question_analysis.php? test_id=1763&package_id=1276&compare_with=3&qid=22859&et1=1&et2=9)

b  Incorrect  :­0.67 Marks  Time:12 Sec.

Rivers of the Chhotanagpur Plateau region have a drainage of trellis pattern. Dendritic Pattern ­ This is a tree­shaped drainage pattern. This is associated with areas of uniform lithology where faulting and jointing are insignificant. Rivers of Indo­Gangetic Plains are of dendritic type. Radial ­ It is pattern characterised by outflowing rivers, away from a central point. Flanks of a dome or a volcanic cone favour the development of ideal radial pattern. Rivers originating from Amarkantak Hills is a good example. Parallel ­ Rivers flow almost parallel to each other and follow the regional slope. Rivers of western coastal plains draining into Arabian Sea is a good example. Annular ­ This is formed when the tributaries of the master consequent streams are developed in the form of a circle. Here, streams follow curving or arcuate courses prior to joining the consequent stream. Nilgiri Hills in Tamil Nadu and Kerala is an example. Trellis ­ Here, the rivers form a net like system and the tributaries flow roughly parallel to each other. Rivers of the Chhotanagpur Plateau region have a drainage of trellis pattern.

a Correct :2 Marks  Time:22 Sec.

Only Manas and Subansiri are the tributaries of Brahmaputra meeting with it in India. Teesta is a tributary of Brahmaputra draining into it in Bangladesh. Barak is not a direct tributary of Brahmaputra. It merges

Select the correct answer using the code give below.   (a) 1 and 3 only (b) 1 and 2 only (c) 3 only (d) 3 and 4 only 

Consider the following rivers: 1.  Manas 2.  Subansiri 3.  Teesta Which of the above tributaries merge with Brahmaputra in India?  56

a Difficult Factual(F) Detailed Analysis (question_analysis.php? test_id=1763&package_id=1276&compare_with=3&qid=22858&et1=1&et2=9)

(a) 1 and 2 only

http://visionias.in/student_open/pt/pta_form.php?test_id=1763&package_id=1276&compare_with=3

Attempted

Guessed between two/ eliminated two Guessed between three/ eliminated one Blind Guess Knew the answer / solved the question

Attempted

Guessed between two/ eliminated two Guessed between three/ eliminated one 31/56

1/25/2016

Vision IAS Prelims

(b) 2 and 3 only

with Padma in Bangaldesh.

(c) 1 and 3 only (d) 1, 2 and 3

The special provisions in Article 371­B of the Constitution of India relate to  (a) Assam (b) Nagaland (c) Manipur 57

a

not attempted

Medium Factual(F) Detailed Analysis (question_analysis.php? test_id=1763&package_id=1276&compare_with=3&qid=22857&et1=1&et2=9)

Article 371 : Maharshtra and Gujarat Article 371­A: Nagaland Article 371­B: Assam Article 371­C: Manipur Article 371­D and Article 371­E: Andhra Pradesh Article 371­F: Sikkim Article 371­G: Mizoram Article 371­H: Arunachal Pradesh Article 371­I: Goa Article 371­J: Karnataka

(d) Andhra Pradesh   

The sales tax you pay while purchasing a newspaper is a (a) tax levied, collected and retained by the centre. (b) tax levied by the centre but collected and appropriated by the states.

c Medium Factual(F) Detailed Analysis (question_analysis.php? test_id=1763&package_id=1276&compare_with=3&qid=22856&et1=1&et2=9)

(c) tax levied and collected by the centre but assigned to the states. (d) tax levied and collected and retained by States  

58

http://visionias.in/student_open/pt/pta_form.php?test_id=1763&package_id=1276&compare_with=3

Article 371 to 371­J of the constitution contain special provisions for eleven states:

not attempted

Following taxes are levied and collected by the Centre but assigned to the States: (a) duties in respect of succession to property other than agricultural land; (b) estate duty in respect of property other than agricultural land; (c) terminal taxes on goods or passengers carried by railway, sea or air; (d) taxes on railway fares and freights; (e) taxes other than stamp duties on transactions in stock exchanges and futures markets; (f) taxes on the sale or purchase of newspapers and on advertisements published therein; (g) taxes on the sale or purchase of goods other than newspapers, where such sale or purchase takes place in the course of inter State trade or commerce; (h) taxes on the consignment of goods (whether the consignment

Blind Guess Knew the answer / solved the question

Unattempted

Absolutely no idea / Could not comprehend the question Not sure of the right answer Inability to recall technique/method Fatigue: not able to concentrate Guessed incorrectly

Unattempted

Absolutely no idea / Could not comprehend the question Not sure of the right answer Inability to recall technique/method Fatigue: not able to concentrate Guessed incorrectly

32/56

1/25/2016

Vision IAS Prelims

is to the person making it or to any other person), where such consignment takes place in the course of inter State trade or commerce

Consider the following: 1.  Right to education 2.  Right to property 3.  Right to freedom of peaceful assembly and association.

59

c Medium Analytical(A) Detailed Analysis (question_analysis.php? test_id=1763&package_id=1276&compare_with=3&qid=22855&et1=1&et2=9)

c Correct :2 Marks  Time:28 Sec.

Right to property was earlier a Fundamental Right dealt by Articles 19(1)(f) and 31 but 44th Amendment Act of 1978 abolished it and inserted a new Article 300A in Part XII under the heading Right to Property. Therefore, it is now a legal/constitutional right and not a Fundamental Right but it is a Human Right under "Universal Declaration of Human Rights". Rest are Fundamental Rights in India as well as Human Rights under the "Universal Declaration of Human Rights .

c  Incorrect  :­0.67 Marks  Time:22 Sec.

Statement 1 is correct: The scheme aims at encouraging States to adequately empower Panchayats and put in place systems for bringing about accountability to PRIs. Statement 2 is incorrect : Panchayat Empowerment and Accountability Incentive Scheme (PEAIS) is a Central Sector plan Scheme being implemented by the Ministry of Panchayati Raj (MoPR) since 2005­06. Extra information: Incentive grants are provided to States under the scheme for undertaking legislative and administrative measures for effective devolution of 3Fs (Function, Fund and Functionaries) to PRIs. PRIs are a part of eleventh

Which of the above Human Right/s under "Universal Declaration of Human Rights" is/are part of Fundamental Rights under Indian Constitution? (a) 1 only (b) 2 and 3 only (c) 1 and 3 only (d) 1 ,2 and 3

With reference to the Panchayat Empowerment & Accountability Incentive Scheme (PEAIS), consider the following statements: 1.  It seeks to empower Panchayats and put in place accountability systems to make their functioning transparent and efficient. 2.  It was launched during 11th Five Year Plan by the Ministry of Rural Development. 

a Medium Factual(F) Detailed Analysis (question_analysis.php? test_id=1763&package_id=1276&compare_with=3&qid=22854&et1=1&et2=9)

Which of the statements given above is/are correct? 60

(a) 1 only (b) 2 only (c) Both 1 and 2 (d) Neither 1 nor 2

http://visionias.in/student_open/pt/pta_form.php?test_id=1763&package_id=1276&compare_with=3

Attempted

Guessed between two/ eliminated two Guessed between three/ eliminated one Blind Guess Knew the answer / solved the question

Attempted

Guessed between two/ eliminated two Guessed between three/ eliminated one Blind Guess Knew the answer / solved the question

33/56

1/25/2016

Vision IAS Prelims

schedule of the Indian constitution and not twelfth schedule.

Consider the following pairs: National Park

 

River on which it is located

1. Anshi           

:

Kali River

2. Kaziranga     

:

Manas River

3. Silent Valley

:

Periyar River

a

not attempted

Difficult Factual(F) Detailed Analysis (question_analysis.php? test_id=1763&package_id=1276&compare_with=3&qid=22853&et1=1&et2=9)

61

Anshi national park of Krantaka is located on the banks of river Kali. It is located in Uttara Kannada district of Karnataka. Kaziranga national park is located in Brahmaputra river not Manas. Silent valley national park is not located on Periyar river. It is located in the Nilgiri Hills, within the Palakkad District of Kerala. The Kuntipuzha River drains the entire 15 km length of the park from north to south into the Bharathapuzha River. 

Unattempted

Absolutely no idea / Could not comprehend the question Not sure of the right answer Inability to recall technique/method Fatigue: not able to concentrate Guessed incorrectly

Which of the pairs given above is/are correct? (a) 1 only (b) 1 and 2 only (c) 2 and 3 only (d) 1, 2 and 3

Consider the following statements with regard to India State of Forest Report (ISFR) 2015:

62

1.  Maximum increase in forest cover has been observed in Very Dense Forest followed by open forests. 2.  Compared to 2013, there is a decrease in the mangrove cover in 2015. 3.  Maximum increase in forest cover has been observed in Tamil Nadu.

c Difficult Factual(F) Detailed Analysis (question_analysis.php? test_id=1763&package_id=1276&compare_with=3&qid=22852&et1=1&et2=9)

Which of the statements given above is/are correct? (a) 1 and 3 only (b) 1 and 2 only

not attempted

Statement 1 is incorrect. As per the India State of Forest Report (ISFR) 2015, maximum increase in forest cover has been observed in open forests followed by very dense forest. Statement 2 is incorrect. Compared to 2013, there is an increase of 112 sq km. in the mangrove cover in 2015. Statement 3 is correct. Maximum increase in forest cover has been observed in Tamil Nadu, followed by Kerala.

Unattempted

Absolutely no idea / Could not comprehend the question Not sure of the right answer Inability to recall technique/method Fatigue: not able to concentrate Guessed incorrectly

(c) 3 only (d) 2 and 3 only http://visionias.in/student_open/pt/pta_form.php?test_id=1763&package_id=1276&compare_with=3

34/56

1/25/2016

Vision IAS Prelims

Consider the following statements: 1.  Balance of Trade is concerned with transactions on account of exports and imports of goods and services. 2.  The responsibility of maintaining Balance of Payment account in India lies with the Department of Economic Affairs, Ministry of Finance. 63

d Medium Factual(F) Detailed Analysis (question_analysis.php? test_id=1763&package_id=1276&compare_with=3&qid=22851&et1=1&et2=9)

c  Incorrect  :­0.67 Marks  Time:2 Sec.

Balance of trade account records all transactions of foreign currencies on account of export and import of goods only.The imports and exports of services are maintained in Balance of Invisibles account. Hence, statement 1 is wrong. The responsibility of maintaining BOP in India lies with Reserve Bank of India. Hence, statement 2 is wrong.

not attempted

Option 1 is incorrect. When RBI follows expansionary monetary policy, it tries to infuse money into market. Thus it will reduce the interest rates in the market. Option 2 is correct. Increased money supply is likely to cause increase in inflation rates. Option 3 is incorrect. Increase in money supply may induce growth and cause higher employment levels.

Which of the statements given above is/are correct? (a) 1 only (b) 2 only (c) Both 1 and 2

Attempted

Guessed between two/ eliminated two Guessed between three/ eliminated one Blind Guess Knew the answer / solved the question

(d) Neither 1 and 2 

When RBI follows expansionary monetary policy, which of the following is/are likely to happen? 1.  Increased interest rates in the market 2.  Increase in inflation rates 3.  Reduction in employment levels  64

b Medium Analytical(A) Detailed Analysis (question_analysis.php? test_id=1763&package_id=1276&compare_with=3&qid=22850&et1=1&et2=9)

Select the correct answer using the code given below. (a) 1 and 2 only (b) 2 only (c) 1 and 3 only (d) 2 and 3 only

Consider the following statements about corals:

65

1.  They live in a symbiotic relationship with algae. 2.  They mostly thrive in warm tropical oceans. 3.  Western coastal waters provide favorable conditions for their growth. 4.  They have low genetic diversity and are vulnerable to changing climatic conditions.

c Difficult Analytical(A) Detailed Analysis (question_analysis.php? test_id=1763&package_id=1276&compare_with=3&qid=22849&et1=1&et2=9)

Which of the statements given above is/are http://visionias.in/student_open/pt/pta_form.php?test_id=1763&package_id=1276&compare_with=3

a  Incorrect  :­0.67 Marks  Time:3 Sec.

Corals mostly thrive in warm tropical oceans, so the difference in ocean temperature between the western coasts and eastern coasts of continents is critical to their distribution. Western coastal waters tend to be cooler, thereby discouraging coral activity, whereas eastern coastal currents are warmer and thus enhance coral growth

Unattempted

Absolutely no idea / Could not comprehend the question Not sure of the right answer Inability to recall technique/method Fatigue: not able to concentrate Guessed incorrectly

Attempted

Guessed between two/ eliminated two Guessed between three/ eliminated one Blind Guess Knew the answer / solved 35/56

1/25/2016

Vision IAS Prelims

correct?

Corals have low genetic diversity worldwide and long generation times, which together mean that corals are slow to adapt and vulnerable to changing conditions.Hence option C is the correct answer.

(a) 1 and 3 only (b) 2 and 3 only (c) 1, 2 and 4 only (d) 1, 2, 3 and 4 

Which of the following is/are the typical characteristics of Mediterranean Climate?

66

1.  Seasonal shifting of tropical wind and pressure belts. 2.  Precipitation falls in vigorous convective showers that are localized and brief. 3.  The modest annual precipitation falls in winter, summers being virtually rainless. 4.  Clear skies and abundant sunshine are typical, especially in summer.

c Medium Factual(F) Detailed Analysis (question_analysis.php? test_id=1763&package_id=1276&compare_with=3&qid=22848&et1=1&et2=9)

c Correct :2 Marks  Time:11 Sec.

Seasonal shifting of tropical wind and pressure belts, especially ITCZ is characteristic of Tropical savanna. Precipitation falls in vigorous convective showers that are localized and of short duration is typical characteristic of Subtropical desert. Mediterranean Climate having modest annual precipitation falls in winter; summers being virtually rainless and clear skies and abundant sunshine are typical, especially in summer.

not attempted

The mangrove cover of India (2.69% of the global mangrove area and 8% of Asia's mangroves) is home to umbrella species such as the tiger as well as many threatened species such as the River terrapin, Gangetic river dolphin, Estuarine crocodile and Fishing cat. Hence option d is the correct answer.

Select the correct answer using the code given below. (a) 1 and 2 only (b) 2 and 3 only (c) 3 and 4 only

the question

Attempted

Guessed between two/ eliminated two Guessed between three/ eliminated one Blind Guess Knew the answer / solved the question

(d) None of the above

Which of the following species can be found in mangroves? 1.  2.  3.  4.  5.  67

Tiger River terrapin Gangetic river dolphin Estuarine crocodile Fishing cat

d Difficult Factual(F) Detailed Analysis (question_analysis.php? test_id=1763&package_id=1276&compare_with=3&qid=22847&et1=1&et2=9)

Select the correct answer using the code given below. (a) 1 and 3 only (b) 2, 3 and 4 only (c) 1, 2, 4 and 5 only (d) 1, 2, 3, 4 and 5

http://visionias.in/student_open/pt/pta_form.php?test_id=1763&package_id=1276&compare_with=3

Unattempted

Absolutely no idea / Could not comprehend the question Not sure of the right answer Inability to recall technique/method Fatigue: not able to concentrate Guessed incorrectly

36/56

1/25/2016

Vision IAS Prelims

Which of the following is/are correct about National Pharmaceutical Pricing Authority? 1.  It decides the ceiling prices of essential medicines. 2.  It works under the Union Ministry of Chemicals and Fertilizers. 68

c Medium Factual(F) Detailed Analysis (question_analysis.php? test_id=1763&package_id=1276&compare_with=3&qid=22846&et1=1&et2=9)

c Correct :2 Marks  Time:48 Sec.

National Pharmaceutical Pricing Authority (NPPA) under the Union Ministry of Chemical and Fertilizer decides the ceiling prices essential medicines under The Drug (Prices Control) Order. Thus both statements are correct. Hence C is the correct answer.

a  Incorrect  :­0.67 Marks  Time:38 Sec.

G8+5 member countries: U.S., U.K., France, Japan, Germany, Canada, Japan, Russia(now suspended) + China, India, Brazil, South Africa, Mexico.

Select the correct answer using the code given below. (a) 1 only (b) 2 only (c) Both 1 and 2

Attempted

Guessed between two/ eliminated two Guessed between three/ eliminated one Blind Guess Knew the answer / solved the question

(d) Neither 1 nor 2

India is not a member of which among the following? 1.  G8+5 2.  G­20 3.  G4

d Difficult Factual(F) Detailed Analysis (question_analysis.php? test_id=1763&package_id=1276&compare_with=3&qid=22845&et1=1&et2=9)

Select the correct answer using the code given below.  (a) 1 only (b) 2 and 3 only (c) 3 only (d) India is a member of all of them

69

G­20 member countries: The G20 is made up of the finance ministers and central bank governors of 19 countries: Argentina, Australia, Brazil, Canada, China, France, Germany, India, Indonesia, Italy, Japan, Mexico, Russia, Saudi Arabia, South Africa, South Korea, Turkey, the United Kingdom, the United States of America. The remaining seat is held by the European Union, which is represented by the rotating Council presidency and the European Central Bank.

Attempted

Guessed between two/ eliminated two Guessed between three/ eliminated one Blind Guess Knew the answer / solved the question

G4 nations: Informal group of countries which support each other’s bids for permanent seats on the United Nations Security Council. It consists of India, Germany, Japan and Brazil. Other important International Organisations that India is a part of: South Asian Association for Regional Cooperation (SAARC), BRICS, Commonwealth of Nations, United Nations, Mekong– Ganga Cooperation, Asia

http://visionias.in/student_open/pt/pta_form.php?test_id=1763&package_id=1276&compare_with=3

37/56

1/25/2016

Vision IAS Prelims

Cooperation Dialogue: (A pan­ Asian organisation which integrates other organisations like SAARC, ASEAN and GCC), Indian­Ocean Rim Association: Organisation of countries which have a coastline on the Indian Ocean, IBSA, East Asia Summit, Shanghai Cooperation Organisation, BIMSTEC.

With reference to Ganga Gram Yojana, consider the following statements: 1.  It is a scheme launched to develop villages in West Bengal situated along the banks of River Ganga. 2.  The programme makes it mandatory for every household in selected villages to have a functional toilet.

b

not attempted

Medium Factual(F) Detailed Analysis (question_analysis.php? test_id=1763&package_id=1276&compare_with=3&qid=22844&et1=1&et2=9)

Which of the statements given above is/are correct? 70

(a) 1 only (b) 2 only (c) Both 1 and 2 (d) Neither 1 nor 2

Which of the following animal species is not found naturally in India? (a) Spectacled Langur (b) Clouded Leopard (c) Hoolock Gibbon

d Easy Factual(F) Detailed Analysis (question_analysis.php? test_id=1763&package_id=1276&compare_with=3&qid=22843&et1=1&et2=9)

(d) Koala Bear  

http://visionias.in/student_open/pt/pta_form.php?test_id=1763&package_id=1276&compare_with=3

not attempted

Statement 1 is incorrect: The Union Government has launched Ganga Gram Yojana to develop 1600 villages in Uttar Pradesh (UP) situated along the banks of Ganga River. Statement 2 is correct: In these selected villages open drains discharging waste directly into river Ganga will be diverted to sewage treatment facility. It would be mandatory for every household in selected villages to have functional toilets. Under this programme, these selected villages will be developed under the Sichewal model (Village in Punjab) which is based on cooperation of the villagers for water management and waste disposal.

Option (a) The dusky leaf monkey, spectacled langur, or spectacled leaf monkey is found naturally in India. Recently, India and Bangladesh have decided to take concerted conservation efforts to protect the rare spectacled langur along with other primates found in Patharia Hills Reserve Forest on either side of the fenced border along Assam's Karimganj district. Option (b) The clouded leopard (Neofelis nebulosa) is a cat found from the

Unattempted

Absolutely no idea / Could not comprehend the question Not sure of the right answer Inability to recall technique/method Fatigue: not able to concentrate Guessed incorrectly

Unattempted

Absolutely no idea / Could not comprehend the question Not sure of the right answer Inability to recall technique/method Fatigue: not able to concentrate Guessed incorrectly

38/56

1/25/2016

Vision IAS Prelims

Himalayan foothills through mainland Southeast Asia into China, and has been classified as Vulnerable in 2008 by the International Union for Conservation of Nature (IUCN). In India they occur in Assam, northern West Bengal, Sikkim, Arunachal Pradesh, Manipur, Meghalaya, Mizoram, Nagaland, and Tripura.  Option (c) The hoolock gibbons, are two primate species native to eastern Bangladesh, Northeast India and Southwest China. In northeast India, the hoolock is found south of Brahmaputra and east of the Dibang Rivers. Its range extends into seven states covering Arunachal Pradesh, Assam, Manipur, Meghalaya, Mizoram, Nagaland, and Tripura (The seven northeastern states of India). Option (d) Koala Bear is an arboreal herbivorous marsupial native to Australia.

71

Which of the following phenomena may cause rising of air? 1.  Convection currents 2.  Convergence of winds 3.  Eddies

d

not attempted

Easy Analytical(A) Detailed Analysis (question_analysis.php? test_id=1763&package_id=1276&compare_with=3&qid=22842&et1=1&et2=9)

Select the correct answer using the code given below. 72

(a) 1 only (b) 1 and 2 only (c)  3 only (d) 1, 2 and 3 

Consider the following statements:

b

http://visionias.in/student_open/pt/pta_form.php?test_id=1763&package_id=1276&compare_with=3



The wind circulation at the earth's surface around low and high on many occasions is closely related to the wind circulation at higher level. Generally, over low pressure area the air will converge and rise. Over high pressure area the air will subside from above and diverge at the surface. Apart from convergence, some eddies, convection currents, orographic uplift and uplift along fronts cause the rising of air, which is essential for the formation of clouds and precipitation.

Normally, temperature

Unattempted

Absolutely no idea / Could not comprehend the question Not sure of the right answer Inability to recall technique/method Fatigue: not able to concentrate Guessed incorrectly

Attempted

39/56

1/25/2016

Vision IAS Prelims

1.  A long summer night with clear skies and still air is ideal for temperature inversion. 2.  Gravitational force plays a major role for temperature inversion to occur in hilly and mountaneous areas.

Difficult Analytical(A) Detailed Analysis (question_analysis.php? test_id=1763&package_id=1276&compare_with=3&qid=22841&et1=1&et2=9)

Incorrect  :­0.67 Marks  Time:40 Sec.

decreases with increase in elevation. It is called normal lapse rate. At times, the situations is reversed and it is called Inversion of temperature.  Statement 1 is incorrect: A long winter night with clear skies and still air is ideal situation for inversion. The heat of the day is radiated off during the night, and by early morning hours, the earth is cooler than the above air. Statement 2 is correct: Temperature inversion takes place in hills and mountains due to air drainage. Cold air at the hills and mountains, prdoduced during night, flows under the influence of gravity. Being heavy and dense, the cold air acts almost like water and moves down the slope to pile up deeply in pockets and valley bottoms with warm air above. This is called air drainage.

b  Incorrect  :­0.67 Marks  Time:2 Sec.

The National Institute of Wind Energy (NIWE) has recently launched Wind Energy Resource Map of India at 100 meter above ground level (AGL) on online Geographic Information System platform.

Which of the statement given above is/are correct? (a) 1 only (b) 2 only (c) Both 1 and 2 73

(d) Neither 1 nor 2

Consider the following states: 1.  2.  3.  4. 

Andhra Pradesh Gujarat Rajasthan Madhya Pradesh

a Difficult Analytical(A) Detailed Analysis (question_analysis.php? test_id=1763&package_id=1276&compare_with=3&qid=22840&et1=1&et2=9)

Arrange the above states in descending order according to their wind energy potential. (a) 2 ­ 1 ­ 3 ­ 4 (b) 2 ­ 3 ­ 1 ­ 4 (c) 1 ­ 2 ­ 3 ­ 4 (d) 1 ­ 2 ­ 4 ­ 3 74

Applying the logic of relation between coastline and wind potential the correct answer may be arrived at. Gujarat has the longest coastline and thus the highest potential and Rajasthan and Madhya Pradesh has none in the given options, thus these two states will come in last among the four states. Only option 'A' satisfies these conditions. 

Guessed between two/ eliminated two Guessed between three/ eliminated one Blind Guess Knew the answer / solved the question

Attempted

Guessed between two/ eliminated two Guessed between three/ eliminated one Blind Guess Knew the answer / solved the question

The State­wise wind energy potential is given below: State Potential (MW@100m) http://visionias.in/student_open/pt/pta_form.php?test_id=1763&package_id=1276&compare_with=3

40/56

1/25/2016

Vision IAS Prelims

Gujarat 84431 Karnataka 55857 Maharashtra 45394 Andhra Pradesh 44229 Tamil Nadu 33800 Rajasthan 18770 Madhya Pradesh 10484 

With reference to Indigo revolt of 1859­60, consider the following statements: 1.  There was unity among Hindu and Muslim peasants during the revolt.  2.  The revolt was successful in spite of violent suppression by British.

a Medium Factual(F) Detailed Analysis (question_analysis.php? test_id=1763&package_id=1276&compare_with=3&qid=22839&et1=1&et2=9)

c  Incorrect  :­0.67 Marks  Time:1 Sec.

The Indigo revolt not only saw unity among Hindu and Muslim peasants, there was also immense role played by intelligentsia of Bengal. Newspaper campaigns, mass meetings, support in legal battles etc. was provided to the peasants by intelligentsia of Bengal, for e.g. Hindoo Patriot published regular reports on planter's oppression. Hence, statement 1 is wrong. The Indigo revolt was largely successful with government issuing notification in 1860 that ryots cannot be compelled to sow indigo. However, the revolt did not see violent repression by the government. The government's response to the revolt was rather restrained, influenced by the support extended to the revolt by the intelligentsia and missionaries. Also, the government has just undergone harrowing experience of Santhal uprising and the revolt of 1857. Hence, Statement 2 is wrong.

c Correct :2 Marks  Time:30 Sec.

Caspian Sea and Black Sea lie to the south west of Russia. Bering Sea lies at the North East of Russia. Yellow Sea is the name given to the northern part of the East China Sea, which is a marginal sea of the Pacific Ocean. It is

Which of the statements given above is/are correct?  (a) 1 only (b) 2 only (c) Both 1 and 2 (d) Neither 1 and 2

75

Which of the following seas does not border Russia? (a) Caspian Sea (b) Black Sea (c) Yellow Sea

c Medium Factual(F) Detailed Analysis (question_analysis.php? test_id=1763&package_id=1276&compare_with=3&qid=22838&et1=1&et2=9)

(d) Bering Sea 76 http://visionias.in/student_open/pt/pta_form.php?test_id=1763&package_id=1276&compare_with=3

Attempted

Guessed between two/ eliminated two Guessed between three/ eliminated one Blind Guess Knew the answer / solved the question

Attempted

Guessed between two/ eliminated two Guessed between three/ eliminated one 41/56

1/25/2016

Vision IAS Prelims

located between mainland China and the Korean Peninsula and its name comes from the sand particles from Gobi Desert sand storms that turn the surface of the water golden yellow.

Consider the following pairs of protected area networks and respective states: 1. Sonai Rupai

:

Arunachal Pradesh

2. Kaibul Lamjao

:

Mizoram

3. Manas

:

Assam

c Difficult Factual(F) Detailed Analysis (question_analysis.php? test_id=1763&package_id=1276&compare_with=3&qid=22837&et1=1&et2=9)

d  Incorrect  :­0.67 Marks  Time:2 Sec.

Statement 1 is incorrect: Sonai Rupai Wildlife Sanctuary is a protected area located in the state of Assam in India. It is located along the foothills of the Great Himalayan Range. Statement 2 is incorrect: The Keibul Lamjao National Park is a national park in the Bishnupur district of the state of Manipur in India. It is the only floating park in the world, located in North East India, and an integral part of Loktak Lake. Statement 3 is correct: Manas National Park or Manas Wildlife Sanctuary is a National Park, UNESCO Natural World Heritage site, a Project Tiger Reserve, an Elephant Reserve and a Biosphere Reserve in Assam, India.

not attempted

Statement 1 is incorrect as sandstone is converted into Quartzite after metamorphism and not schist. Statements 2, 3 and 4 are correct as limestone is converted into marble while Granite into Gneiss and shale into slate after the process of metamorphism.

77 Which of the pairs given above is/are correctly matched? (a) 1 and 3 only (b) 2 only (c) 3 only (d) 1, 2 and 3

Consider the following pairs:

78

Original Rock

 

Metamorphic Rock

1. Sandstone

:

Schist

2. Limestone

:

Marble

3. Granite

:

Gneiss

c Difficult Factual(F) Detailed Analysis (question_analysis.php? test_id=1763&package_id=1276&compare_with=3&qid=22836&et1=1&et2=9)

http://visionias.in/student_open/pt/pta_form.php?test_id=1763&package_id=1276&compare_with=3

Blind Guess Knew the answer / solved the question

Attempted

Guessed between two/ eliminated two Guessed between three/ eliminated one Blind Guess Knew the answer / solved the question

Unattempted

Absolutely no idea / Could not comprehend the question Not sure of the right answer Inability to recall technique/method Fatigue: not able to concentrate Guessed incorrectly

42/56

1/25/2016

Vision IAS Prelims

4. Shale

:

Slate

Which of the pairs given above is/are correctly matched? (a) 1 and 2 only (b) 1, 3 and 4 only (c) 2, 3 and 4 only (d) 2 and 4 only

With reference to the Pitt's India Act of 1784, consider the following statements: 1.  It distinguished between the commercial and political functions of the Company. 2.  It designated the Governor of Bengal as the 'Governor­General of Bengal' 3.  It attempted to introduce a system of open competition for selection of civil servants. 79

a

not attempted

Medium Factual(F) Detailed Analysis (question_analysis.php? test_id=1763&package_id=1276&compare_with=3&qid=22835&et1=1&et2=9)

Which of the statements given above is/are correct? (a) 1 only (b) 1 and 2 only (c) 2 only

Statement 1 is correct: It distinguished between the commercial and political functions of the Company. Statement 2 is incorrect: Regulating Act of 1773 designated the Governor of Bengal as the 'Governor­ General of Bengal'. Statement 3 is incorrect: The Charter Act of 1833 attempted to introduce a system of open competition for selection of civil servants. Hence A is the correct answer.

Unattempted

Absolutely no idea / Could not comprehend the question Not sure of the right answer Inability to recall technique/method Fatigue: not able to concentrate Guessed incorrectly

(d) 2 and 3 only

Which of the following discretionary powers is/are provided to the President under the Indian Constitution? 1.  Ordinance making power 2.  Appointment of Prime Minister when no party has a clear majority 3.  Dissolution of the Lok Sabha if the council of ministers has lost its majority 4.  Pardoning power

d Medium Analytical(A) Detailed Analysis (question_analysis.php? test_id=1763&package_id=1276&compare_with=3&qid=22834&et1=1&et2=9)

Select the correct answer using the code given below. 80

(a) 1 only (b) 2 and 3 only (c) 1 and 4 only

b  Incorrect  :­0.67 Marks  Time:5 Sec.

The President has no constitutional discretion, he has some situational discretion. In other words, the President can act on his discretion (that is, without the advice of the ministers) under the following situations: (i) Appointment of Prime Minister when no party has a clear majority in the Lok Sabha or when the Prime Minister in office dies suddenly and there is no obvious successor.

Attempted

Guessed between two/ eliminated two Guessed between three/ eliminated one Blind Guess Knew the answer / solved the question

(ii) Dismissal ofthe council ofministers when it cannot prove the confidence ofthe Lok Sabha.

(d) None http://visionias.in/student_open/pt/pta_form.php?test_id=1763&package_id=1276&compare_with=3

43/56

1/25/2016

Vision IAS Prelims

(iii) Dissolution ofthe Lok Sabha ifthe council ofministers has lost its majority. Both ordinance making power and emergency powers are not discretionary powers as president acts upon the advice of the council of minister. Hence 'D' is the correct answer.

Which of the following modifications occur/occurs in hydrophytes? 1.  Flexible leaves and stem 2.  Reduced roots and root hair 3.  Air spaces in stem

d

not attempted

Medium Factual(F) Detailed Analysis (question_analysis.php? test_id=1763&package_id=1276&compare_with=3&qid=22833&et1=1&et2=9)

1.  Underwater leaves and stems are flexible to move with water currents 2.  Some plants have air spaces in their stems to help hold the plant up in the water 3.  Submerged plants lack strong water transport system (in stems); instead water, nutrients, and dissolved gases are absorbed through the leaves directly from the water. 4.  Roots and root hairs reduced or absent; roots only needed for anchorage, not for absorption of nutrients and water 5.  Some plants have leaves that float atop the water, exposing themselves to the sunlight in floating plants chlorophyll is restricted to upper 6.  Surface of leaves (part that the sunlight will hit) and the upper surface is waxy to repel water 7.  Some plants produce seeds that can float

Select the correct answer using the code given below. (a) 1 and 2 only  (b) 2 only (c) 1 and 3 only (d) 1, 2 and 3

81

The unique faunal and floristic developments of Australia are primarily due to which of the following reasons? (a) It is isolated from rest of the continents (b) It is located in Oceania between the Indian Ocean and the South Pacific Ocean

a Difficult Analytical(A) Detailed Analysis (question_analysis.php? test_id=1763&package_id=1276&compare_with=3&qid=22832&et1=1&et2=9)

http://visionias.in/student_open/pt/pta_form.php?test_id=1763&package_id=1276&compare_with=3

Plant Adaptations in Water/ Adaptations of Hydrophytes:

not attempted

The unique biota of Australia is largely a result of isolation. During long periods of time in the geologic past, while most of the continents were connected, the climate was more equitable, allowing for

Unattempted

Absolutely no idea / Could not comprehend the question Not sure of the right answer Inability to recall technique/method Fatigue: not able to concentrate Guessed incorrectly

Unattempted

Absolutely no idea / Could not comprehend the question Not sure of the 44/56

1/25/2016

Vision IAS Prelims

(c) It has a unique combination of various landforms 82

a vast evolution of species. As the continents began to separate about two hundred million years ago, climatic changes occurred requiring new adaptation and continued evolution. Australia became isolated from the rest of the continents and so evolution continued with little genetic influence from the outside. This has engendered specialized evolutionary development among the isolated flora and fauna of Australia.

(d) It is uniquely placed between 10°S and 40°S

Consider the following statements: 1.  Lunar tides are about twice as strong as the solar tides 2.  The magnitude of tidal fluctuation is similar everywhere on earth 3.  On a given day, there is normally one high tide and one low tide

a Medium Analytical(A) Detailed Analysis (question_analysis.php? test_id=1763&package_id=1276&compare_with=3&qid=22831&et1=1&et2=9)

a Correct :2 Marks  Time:15 Sec.

Which of the above statements regarding tides is/are correct? 83

(a) 1 only (b) 1 and 2 only (c) 3 only (d) 1, 2 and 3

With reference to public debt of India, consider the following statements: 1.  It consists of internal debt, external debt and public account liabilities. 2.  In India, the share of internal debt forms a higher proportion of public debt in India than external debt. 84

b Difficult Factual(F) Detailed Analysis (question_analysis.php? test_id=1763&package_id=1276&compare_with=3&qid=22830&et1=1&et2=9)

Which of the statements given above is/are correct?  (a) 1 only (b) 2 only

http://visionias.in/student_open/pt/pta_form.php?test_id=1763&package_id=1276&compare_with=3

a  Incorrect  :­0.67 Marks  Time:21 Sec.

The strength of the force of gravity is inversely proportional to the square of the distance between the two bodies. Therefore, the Sun being 93,000,000 miles away from Earth and the Moon 239,000, the lunar tides are about twice as strong as the solar tides. The magnitude of tidal fluctuation is quite variable in time and place, but the sequence of the cycle is generally similar everywhere. The tides rise and fall twice in the interval of 24 hours and 50 minutes. Therefore on all coastlines there are normally two high tides and two low tides.

In India, total Central Government Liabilities constitutes the following three categories; Internal Debt. External Debt. Public Account Liabilities. However, Public Debt in India includes only Internal and External Debt incurred by the Central Government. Hence, Statement 1 is wrong.

right answer Inability to recall technique/method Fatigue: not able to concentrate Guessed incorrectly

Attempted

Guessed between two/ eliminated two Guessed between three/ eliminated one Blind Guess Knew the answer / solved the question

Attempted

Guessed between two/ eliminated two Guessed between three/ eliminated one Blind Guess Knew the answer / solved the question 45/56

1/25/2016

Vision IAS Prelims

(c) Both 1 and 2

Internal debt of the Central Government constituted 92.9 per cent of public debt for 2014­15. Hence, statement 2 is correct.

(d) Neither 1 and 2

With reference to the 'Mansabdari System' during the Mughals, consider the following statements: 1.  It was a system of giving rank/position only to the soldiers and officials of the Army. 2.  The 'Mansabdars' received their salaries only in the form of jagirs.

d Medium Factual(F) Detailed Analysis (question_analysis.php? test_id=1763&package_id=1276&compare_with=3&qid=22829&et1=1&et2=9)

c  Incorrect  :­0.67 Marks  Time:24 Sec.

Statement 1 is incorrect: The whole nobility, the bureaucracy as well as the military hierarchy, held mansabs. They could be transferred from the civil side to the military department and vice versa. Statement 2 is incorrect: The mansabdars of the Mughal empire received their pay either in cash (naqd) or in the form of assignments of areas of land (jagir) from which they were entitled to collect the land revenue and all other taxes sanctioned by the emperor.  The unique feature of Mansabdari was Tajwiz System', wherein a petition was by a nobleman to the Emperor to recommend an applicant to be recruited as a 'mansabdar'.

b  Incorrect  :­0.67 Marks  Time:28 Sec.

Statement 1 is correct : It originated during Post Mauryan times. Statement 2 is correct: It believed in doctrine of reincarnation or 'avataras'. Statement 3 is incorrect: It was liberal enough to attract foreigners into its fold along with women,vaishyas and shudras .

Which of the statements given above is/are correct?  (a) 1 only 85

(b) 2 only (c ) Both 1 and 2 (d) Neither 1 nor 2

With reference to Bhagvatism cult, consider the following statements: 1.  It originated during Post Mauryan period. 2.  It believed in the doctrine of reincarnation. 3.  It did not allow women and shudras to become part of the cult.  86

a Difficult Factual(F) Detailed Analysis (question_analysis.php? test_id=1763&package_id=1276&compare_with=3&qid=22828&et1=1&et2=9)

Which of the statements given above is/are correct? (a) 1 and 2 only (b) 2 only

Attempted

Guessed between two/ eliminated two Guessed between three/ eliminated one Blind Guess Knew the answer / solved the question

Attempted

Guessed between two/ eliminated two Guessed between three/ eliminated one Blind Guess Knew the answer / solved the question

(c) 1, 2 and 3 only (d) None   

Attempted

http://visionias.in/student_open/pt/pta_form.php?test_id=1763&package_id=1276&compare_with=3

46/56

1/25/2016

Vision IAS Prelims

Who among the following was/were not an Extremist leader? 1.  2.  3.  4.  87

Rash Behari Ghose Gopal Krishna Gokhale  Rash Behari Bose Aurbindo Ghosh

a Medium Factual(F) Detailed Analysis (question_analysis.php? test_id=1763&package_id=1276&compare_with=3&qid=22827&et1=1&et2=9)

d  Incorrect  :­0.67 Marks  Time:53 Sec.

Moderate leaders: Rash Behari Ghose and Gopal Krishna Gokhale Extremist leaders: Rash Behari Bose and Aurbindo Ghosh

a Correct :2 Marks  Time:2 Sec.

Statement 1 is correct: In case of dissolution of Panchayats before expiry of its term, it is mandatory to hold elections within 6 months of its dissolution. Statement 2 is incorrect : Every Panchayat shall continue for full term of 5 years. However, a Panchayat reconstituted after premature dissolution shall continue for only the reminder of the period. If the period is less than 6 months then it is not mandatory to held the elections. Statement 3 is incorrect : A person who is otherwise eligible for election to state legislature is eligible for Panchayats elections provided he has attained 21 yrs of age and not 25 years as applicable for State legislature.

d  Incorrect  :­0.67 Marks  Time:2 Sec.

Statement 1 is incorrect: Election Commission of India is a permanent Constitutional Body. The Election Commission was established in accordance with the Constitution on 25th January 1950.

Select the correct answer using the code given below. (a) 1 and 2 only (b) 1, 2 and 4 only (c) 2 and 3 only

Attempted

Guessed between two/ eliminated two Guessed between three/ eliminated one Blind Guess Knew the answer / solved the question

(d) 3 and 4 only

With reference to Panchayati Raj Institutions (PRIs), consider the following statements: 1.  In case of dissolution of panchayats before expiry of its term, it is mandatory to hold elections within 6 months of its dissolution. 2.  A panchayat reconstituted after premature dissolution shall continue for full 5 years. 3.  All persons who have attained minimum 25 years are eligible for elections in panchayats. 88

a Medium Factual(F) Detailed Analysis (question_analysis.php? test_id=1763&package_id=1276&compare_with=3&qid=22826&et1=1&et2=9)

Which of the statements given above is/are correct?  (a) 1 only (b) 1 and 3 only (c) 2 and 3 only (d) None 

With reference to the Election Commission of India, consider the following statements: 1.  Election Commission of India is a non­permanent constitutional body. 2.  The Secretariat of the Commission has an independent budget. 3.  The decisions of the Election Commission can only be challenged

b Difficult Factual(F) Detailed Analysis (question_analysis.php? test_id=1763&package_id=1276&compare_with=3&qid=22825&et1=1&et2=9)

http://visionias.in/student_open/pt/pta_form.php?test_id=1763&package_id=1276&compare_with=3

Attempted

Guessed between two/ eliminated two Guessed between three/ eliminated one Blind Guess Knew the answer / solved the question

Attempted

Guessed between two/ eliminated two Guessed between three/ 47/56

1/25/2016

Vision IAS Prelims

in the Supreme Court.

Statement 2 is correct: The Secretariat of the Commission has an independent budget, which is finalised directly in consultation between the Commission and the Finance Ministry of the Union Government. The latter generally accepts the recommendations of the Commission for its budgets. Statement 3 is incorrect: The decisions of the Commission can be challenged in the High Court and the Supreme Court of the India by appropriate petitions. By long standing convention and several judicial pronouncements, once the actual process of elections has started, the judiciary does not intervene in the actual conduct of the polls. Once the polls are completed and result declared, the Commission cannot review any result on its own. This can only be reviewed through the process of an election petition, which can be filed before the High Court, in respect of elections to the Parliament and State Legislatures. In respect of elections for the offices of the President and Vice President, such petitions can only be filed before the Supreme Court.

Which of the statements given above is/are correct ?  (a) 1, 2 and 3 (b) 2 only (c) 2 and 3 only (d) None 

89

Which of the following does not relate to the Fundamental Rights as enshrined in the Constitution? 1.  The right against tapping of telephonic conversation. 2.  Free and compulsory education to all children of the age of 6­14 years. 3.  The right to obtain recognition of the association. 4.  Promotion of the educational and

d Medium Factual(F) Detailed Analysis (question_analysis.php? test_id=1763&package_id=1276&compare_with=3&qid=22824&et1=1&et2=9)

http://visionias.in/student_open/pt/pta_form.php?test_id=1763&package_id=1276&compare_with=3

b  Incorrect  :­0.67 Marks  Time:20 Sec.

Statement 1 is a Fundamental Right: The right against tapping of telephonic conversation is provided under Art.19(a)as part of Freedom of Speech and Expression. Statement 2 is a Fundamental Right: Art 21­ A­ Free and compulsory education to all children of

eliminated one Blind Guess Knew the answer / solved the question

Attempted

Guessed between two/ eliminated two Guessed between three/ eliminated one Blind Guess Knew the 48/56

1/25/2016

Vision IAS Prelims

economic interests of the weaker section of the people, especially the Scheduled Castes and Scheduled Tribes.

90

the age of 6­14 years was inserted by 86th Amendment Act, 2002. Statement 3 is not a Fundamental Right: Although citizens have the right to form associations and unions under Art. 19(c). However,the right to obtain recognition of th association is not a fundamental right. Statement 4 is not a Fundamental Right: It is a part of Directive Principles of State Policy( Art. 46).

Select the correct answer using the code given below. (a) 1 and 2 only (b) 1 and 4 only (c) 1, 3 and 4 only (d) 3 and 4 only

Consider the following towns: 1.  2.  3.  4. 

Poochampally Paithan Sanganer Kota

a Very difficult Factual(F) Detailed Analysis (question_analysis.php? test_id=1763&package_id=1276&compare_with=3&qid=22823&et1=1&et2=9)

Which of the above are famous for the production of traditional sarees/fabrics? (a) 1, 2 and 4 only (b) 2 and 3 only (c) 1 and 4 only (d) 1, 2, 3 and 4

91

http://visionias.in/student_open/pt/pta_form.php?test_id=1763&package_id=1276&compare_with=3

not attempted

Poochampally: Pochampally Saree or Pochampalli Ikat is a saree made in Bhoodan Pochampally, Nalgonda district, Telangana State, India. They are popular for their traditional geometric patterns in Ikat style of dyeing. The Indian government's official air carrier, Air India air hostesses wear specially designed pochampally silk sarees.It has found place in UNESCO tentative list of world heritage sites as part of "iconic saree weaving clusters of India". Paithani: Paithani is a variety of sari, named after the Paithan town in Aurangabad Maharashtra state where they are woven by hand. Made from very fine silk, it is considered as one of the richest saris in India, it is a gold and silk saree( zari ). Paithani is characterised by borders of an oblique square design, and a pallu with a peacock design. Sanganer: a town near Jaipur, is famous for block and screen­printed cotton cloth. In the traditional Sanganeri prints, the ground is in white or pastel

answer / solved the question

Unattempted

Absolutely no idea / Could not comprehend the question Not sure of the right answer Inability to recall technique/method Fatigue: not able to concentrate Guessed incorrectly

49/56

1/25/2016

Vision IAS Prelims

shades with floral cones and sprays scattered with in symmetrical borders. Sanganeri Hand block printing received the geographical indication (GI) tag in 2010. Kota: in Rajasthan, famous for Kota doria or Kota Saris. Sarees are made of pure cotton and silk and have square like patterns known as khats on them. The chequered weave of a Kota sari is very popular. They are very fine weaves and weigh very less.

Consider the following pairs: Dance

 

State

Krishanattam

:

Kerela

Koodiyaattam

:

Andhra Pradesh

Mohiniattam

:

Orissa

a Medium Factual(F) Detailed Analysis (question_analysis.php? test_id=1763&package_id=1276&compare_with=3&qid=22822&et1=1&et2=9)

Which of the above pairs given above is/are correctly matched? (a) 1 only (b) 1 and 3 only (c) 2 and 3 only (d) 1, 2 and 3

92

http://visionias.in/student_open/pt/pta_form.php?test_id=1763&package_id=1276&compare_with=3

b  Incorrect  :­0.67 Marks  Time:3 Sec.

All the three belong to Kerela. Krishnattam, folk theatre of Kerala, came into existence in the middle of 17th century A.D. under the patronage of King Manavada of Calicut. Krishnattam is a cycle of eight plays performed for eight consecutive days. The plays are Avataram, Kaliamandana, Rasa krida, kamasavadha, Swayamvaram, Bana Yudham, Vivida Vadham, and Swargarohana. The episodes are based on the theme of Lord Krishna ­ his birth, childhood pranks and various deeds depicting victory of good over evil. Koodiyaattam, one of the oldest traditional theatre forms of Kerala, is based on Sanskrit theatre traditions. The characters of this theatre form are: Chakyaar or actor, Naambiyaar, the instrumentalists and Naangyaar, those taking on women's roles. The Sutradhar or narrator and the Vidushak or jesters are the protagonists.Emphasis on hand gestures and eye movements makes this dance and theatre form unique.

Attempted

Guessed between two/ eliminated two Guessed between three/ eliminated one Blind Guess Knew the answer / solved the question

50/56

1/25/2016

Vision IAS Prelims

Mohiniyattam, is the classical solo dance form of Kerala. It is literally interpreted as the dance of ‘Mohini’, the celestial enchantress of the Hindu mythology. It is one of the 8 classical dance forms of India. References of Mohiniyattam can be found in the texts Vyavaharamala written in 1709 by Mazhamagalam Narayanan Namputiri and in Ghoshayatra, written later by great poet Kunjan Nambiar. It traces its origin to the temples of Kerala.

Match List I with List II and select the correct answer using the code given below the lists:  List I

List II

(Optical phenomena)

 

1. Diffraction 

:

A. Blue color of sky

2. Total Internal Reflection

:

B. Splitting of white light into a rainbow pattern.

3. Scattering 

:

C. Sparkles in diamond

:

D. Rainbow pattern we see when looking at a CD or DVD

4. Dispersion 

a Medium Analytical(A) Detailed Analysis (question_analysis.php? test_id=1763&package_id=1276&compare_with=3&qid=22821&et1=1&et2=9)

(Effect)

(a) 1­D, 2­C, 3­A, 4­B 93

(b) 1­A, 2­C, 3­B, 4­D (c) 1­D, 2­A, 3­C, 4­B

http://visionias.in/student_open/pt/pta_form.php?test_id=1763&package_id=1276&compare_with=3

b  Incorrect  :­0.67 Marks  Time:2 Sec.

Diffraction is defined as the bending of light around the corners of an obstacle or aperture into the region of geometrical shadow of the obstacle. The effects of diffraction are often seen in everyday life. For example, the closely spaced tracks on a CD or DVD act as a diffraction grating to form the familiar rainbow pattern seen when looking at a disk. Total internal reflection (TIR) is a phenomenon which occurs when a propagating wave strikes a medium boundary at an angle larger than a particular critical angle with respect to the normal to the surface. If the refractive index is lower on the other side of the boundary and the incident angle is greater than the critical angle, the wave cannot pass through and is entirely reflected. Due to TIR while swimming, when one opens one's eyes just under the water's surface. If the water is calm, its surface appears mirror­like. Diamonds used for jewelry

Attempted

Guessed between two/ eliminated two Guessed between three/ eliminated one Blind Guess Knew the answer / solved the question

51/56

1/25/2016

Vision IAS Prelims

(d) 1­B, 2­C, 3­A, 4­D

Which of the following factor is primarily responsible for formation of Patagonian desert? (a) Off­shore planetary winds blowing in the mid­latitudes.

are normally shaped in such a way as to maximize the amount of light that is totally internally reflected from the back side of the diamond. Mirage also occurs due to TIR. Optical fibres use the principle of TIR.  Rayleigh scattering is the way that light scatters when it hits a very small object. The theory is good if the wavelength of light is much bigger than the object. The reason for the blue color of the sky and the yellow tone of the sun is scattering. Dispersion is the phenomenon in which the phase velocity of a wave depends on its frequency. In a dispersive prism, material dispersion (a wavelength­dependent refractive index) causes different colors to refract at different angles, splitting white light into a rainbow.

b

not attempted

Medium Factual(F) Detailed Analysis (question_analysis.php? test_id=1763&package_id=1276&compare_with=3&qid=22820&et1=1&et2=9)

(b) Rain shadow position on leeward side of the Andes. 94

(c) Interior location in the continent. (d) Cold oceanic current along the east coast of South America.

Consider the following crops of India: 1.  2.  3.  4. 

Rice Gram Rapeseeds Mustard

b Medium Factual(F) Detailed Analysis (question_analysis.php? test_id=1763&package_id=1276&compare_with=3&qid=22819&et1=1&et2=9)

http://visionias.in/student_open/pt/pta_form.php?test_id=1763&package_id=1276&compare_with=3

a  Incorrect  :­0.67 Marks  Time:24 Sec.

The Patagonian Desert is a large, cold, mid­latitude winter desert. It is primarily a desert due to its rain­ shadow position in the leeward side of lofty Andes than continentality. The Andes inhibit the westerly flow of moisture from the southern Pacific from reaching inland. Other mid­ latitude deserts such as Gobi and Turkestan are formed due to their interior position in the continent away from the rain­bearing winds.

There are three distinct crop seasons in the northern and interior parts of country, namely kharif, rabi and zaid. The kharif season largely coincides

Unattempted

Absolutely no idea / Could not comprehend the question Not sure of the right answer Inability to recall technique/method Fatigue: not able to concentrate Guessed incorrectly

Attempted

Guessed between two/ eliminated two 52/56

1/25/2016

Vision IAS Prelims

5.  Tur 6.  Cotton 95

with Southwest Monsoon under which the cultivation of tropical crops such as rice, cotton, jute, jowar, bajra and tur is possible. The Rabi season begins with the onset of winter in October­November and ends in March­April. Major crops of Rabi are Wheat, Gram, Rapeseeds and Mustard, Barley.

Which of the above are predominantly Rabi crop/crops? (a) 1, 2, 3 and 6 only (b) 2, 3 and 4 only (c) 2, 4 and 5 only (d) 3, 4 and 6 only

Which of the following rivers passes through Mizoram? 1.  2.  3.  4. 

Dihang Barak Kaladan Subansiri

b Difficult Factual(F) Detailed Analysis (question_analysis.php? test_id=1763&package_id=1276&compare_with=3&qid=22818&et1=1&et2=9)

b Correct :2 Marks  Time:18 Sec.

There are a number of rivers which flow through the state. Some of the rivers which are majorly part of the country of Bangladesh but passes through the state of Mizoram also are Thega River and the Surma­ Meghna river system (Barak River). Tiau River, Tlawng, Barak, Kaladan, Karnaphuli are some major rivers of Mizoram. Other rivers and tributaries which flow through the state of Mizoram include Tuichong River, Chhimtuipui River, Ngengpui River, Chawngte River, Tuiphal River, Mat River and Kawrthingdeng River.The Subansiri River is a tributary of the Brahmaputra River in the Indian states of Assam and Arunachal Pradesh. The Manas River is a transboundary river in the Himalayan foothills between southern Bhutan and India.

not attempted

If the buyers of the shares of a particular company are more than the sellers of the same, the traded value of the shares would be at a 'premium' or otherwise it would be at a 'discount'. Thus, in both the cases of

Select the correct answer using the code given below. (a) 1 and 2 only (b) 2 and 3 only (c) 1, 3 and 4 only (d) 1, 2, 3 and 4 96

Consider the following statements: 1.  While the purpose of disinvestment is only to raise resources, privatization involves transferring of management control along with raising resources. 2.  While in the case of disinvestment the shares are sold at discount, in case of

a Difficult Analytical(A) Detailed Analysis (question_analysis.php? test_id=1763&package_id=1276&compare_with=3&qid=22817&et1=1&et2=9)

http://visionias.in/student_open/pt/pta_form.php?test_id=1763&package_id=1276&compare_with=3

Guessed between three/ eliminated one Blind Guess Knew the answer / solved the question

Attempted

Guessed between two/ eliminated two Guessed between three/ eliminated one Blind Guess Knew the answer / solved the question

Unattempted

Absolutely no idea / Could not comprehend the question Not sure of the 53/56

1/25/2016

Vision IAS Prelims

privatization they are sold at premium. 97

disinvestment as well as privatization, selling of shares of public sector would be at a premium. Hence, statement 2 is wrong.

Which of the statements given above is/are correct? (a) 1 only (b) 2 only (c) Both 1 and 2

right answer Inability to recall technique/method Fatigue: not able to concentrate Guessed incorrectly

(d) Neither 1 and 2

Consider the following statements regarding Hepatitis B: 1.  It is an infectious disease which affects the liver. 2.  Most infected persons do not show any symptoms. 3.  There is no vaccination available against it.

b

not attempted

Medium Factual(F) Detailed Analysis (question_analysis.php? test_id=1763&package_id=1276&compare_with=3&qid=22816&et1=1&et2=9)

Which of the statements given above is/are correct? (a) 1 only (b) 1 and 2 only (c) 2 and 3 only (d) 1, 2 and 3

98

Which of the following pairs is/ are correctly matched?

b

http://visionias.in/student_open/pt/pta_form.php?test_id=1763&package_id=1276&compare_with=3

not attempted

Hepatitis B is a viral infection that attacks the liver and can cause both acute and chronic disease. The virus is transmitted through contact with the blood or other body fluids of an infected person. An estimated 240 million people are chronically infected with hepatitis B (defined as hepatitis B surface antigen positive for at least 6 months). More than 780 000 people die every year due to complications of hepatitis B, including cirrhosis and liver cancer1. Hepatitis B is an important occupational hazard for health workers. However, it can be prevented by currently available safe and effective vaccine. The Government of India has included Hepatitis B vaccine under mission Indradhanush. The Mission Indradhanush, depicting seven colours of the rainbow, targets to immunize all children against seven vaccine preventable diseases, namely: Diphtheria, Pertussis (Whooping Cough), Tetanus, Tuberculosis, Polio, Hepatitis B, Measles. 

In WTO terminology, subsidies in general are

Unattempted

Absolutely no idea / Could not comprehend the question Not sure of the right answer Inability to recall technique/method Fatigue: not able to concentrate Guessed incorrectly

Unattempted

54/56

1/25/2016

1. Green Box subsidy

2. Blue Box

:

Subsidies that distort trade.

:

Subsidies that aims to limit production.

:

Subsidies that don’t distort trade.

subsidy

3. Amber Box subsidy:

Vision IAS Prelims Difficult Factual(F) Detailed Analysis (question_analysis.php? test_id=1763&package_id=1276&compare_with=3&qid=22815&et1=1&et2=9)

Select the correct answer using the code given below. (a) 1 and 3 only (b) 2 only (c) 3 only (d) None of the above

99

http://visionias.in/student_open/pt/pta_form.php?test_id=1763&package_id=1276&compare_with=3

identified by "boxes" which are given the colours of traffic lights: green (permitted), amber (slow down ­ i.e. be reduced), red (forbidden). In agriculture, things are, as usual, more complicated. The Agriculture Agreement has no red box, although domestic support exceeding the reduction commitment levels in the amber box is prohibited; and there is a blue box for subsidies that are tied to programmes that limit production. The 'amber box': For agriculture, all domestic support measures considered to distort production and trade (with some exceptions) fall into the amber box. The 'green box': In order to qualify for the "green box", a subsidy must not distort trade, or at most cause minimal distortion. These subsidies have to be government­funded (not by charging consumers higher prices) and must not involve price support. The 'blue box': The blue box is an exemption from the general rule that all subsidies linked to production must be reduced or kept within defined minimal ("de minimis") levels. It covers payments directly linked to acreage or animal numbers, but under schemes which also limit production by imposing production quotas or requiring farmers to set aside part of their land. Countries using these subsidies ­ and there are only a handful ­ say they distort trade less than alternative amber box subsidies. Currently, the only members notifying the

Absolutely no idea / Could not comprehend the question Not sure of the right answer Inability to recall technique/method Fatigue: not able to concentrate Guessed incorrectly

55/56

1/25/2016

Vision IAS Prelims

WTO that they are using or have used the blue box are: the EU, Iceland, Norway, Japan, the Slovak Republic, Slovenia, and the US (now no longer using the box).

The Biodiversity Act ­ 2002 addresses issues of sustainable use of biological resources in the country. Which of the following is/are exempted from prior approval under the act for obtaining biological resources?

d Medium Analytical(A) Detailed Analysis (question_analysis.php? test_id=1763&package_id=1276&compare_with=3&qid=22814&et1=1&et2=9)

1.  Local people and communities of the area. 2.  Vaids and Hakims 3.  Government sponsored collaborative research programs Select the correct answer using the code given below. (a) 1 only (b) 1 and 2 only (c) 2 and 3 only (d) 1, 2 and 3 100

a  Incorrect  :­0.67 Marks  Time:47 Sec.

The legislation provides for the following exemptions a. Exemption to local people and communities of the area for free access to use biological resources within India b. Exemptions to growers and cultivators of biodiversity and to Vaids and Hakims to use biological resources. c. Exemption through notification of normally traded commodities from the purview of the Act only when used as commodity d. Exemption for collaborative research through government sponsored or government approved institutions subject to overall policy guidelines and approval of the Central Government and conforms to the central government guidelines. Hence D is the correct answer. The Biodiversity Act ­ 2002 primarily addresses issues of conservation, sustainable use of biological resources in the country, issue related to access to genetic resources and associated knowledge and fair and equitable sharing of benefits arising from utilization of biological resources to the country and its people.

Attempted

Guessed between two/ eliminated two Guessed between three/ eliminated one Blind Guess Knew the answer / solved the question

Submit and analyse your result (average_marks_over_range_preparation.php?test_id=1763&package_id=1276&compare_with=3)

http://visionias.in/student_open/pt/pta_form.php?test_id=1763&package_id=1276&compare_with=3

56/56

Related Documents


More Documents from "Utkarsh Verma"

Vision Ias Prelims
January 2021 2